47 High Yield Recall

Published on July 2016 | Categories: Documents | Downloads: 67 | Comments: 0 | Views: 620
of 45
Download PDF   Embed   Report

Comments

Content

www.usmle.tv
ANATOMY Ø Drainage pattern to the superficial and deep inguinal lymph nodes. - Anything from the internal genitalia (testicles, ovaries, uterus) drains to the para-aortic lymph nodes - But what drains into the superficial and deep inguinal lymph nodes? - Deep inguinal lymph nodes drain the lower limb, while superficial nodes drain the external genitalia and superficial tissues. This is why we never resect or even biopsy via a transcrotal approach despite the fact that the testicles drain into the para-aortics.

Ø HY TIPS - 1. Direct hernia: leaves abdominal cavity medial to inferior epigastric vessels Indirect hernia: leaves abdominal cavity lateral to inferior epigastric vessels Femoral hernia: protrusion of abdominal viscera through femoral ring into femoral canal Lumbar puncture: needle into lumbar cistern between spinous processes L3/L4 or L4/L5 Pericardiocentesis: wide bore needle inserted through 5th or 6th intercostal space near sternum. Careful not to puncture internal thoracic artery - 2. Thyroid C5 Duodenum T12-L1 Sternal notch T2 Kidneys T12-L3 Bifurcation of trachea T4-T5 Conus medularis L1-L2 adult, L3 newborn Heart: Base T6-T9 Umbilicus L4 Apex 5th left intercostal space - 3. Knee: 1. Patellar ligament- damage to femoral nerve or spinal cord L2-L4. Loss of patellar reflex 2. MCL- tear also tears medial meniscus. Passive abduction of extended leg at knee joint. 3. LCL- passive adduction of extended leg at knee joint. 4. ACL- anterior drawer sign. 5. PCL- posterior drawer sign. 6. Terrible triad- MCL, medial meniscus and ACL tears. - Hip: 1. Posterior dislocation- head of femur moves posterior to the iliofemoral ligament. Presents with lower limb that is flexed at hip joint, adducted, medial rotated and shorter than opposite limb. 2. Fracture of neck of femur presents laterally rotated and shortened. - Shoulder: 1. Dislocation- may be anterior or posterior. If anterior then axillary nerve may be damaged. 2. Separation- results in a downward displacement of clavicle. - Clavicle: 1. Fracture- most common at medial 1/3. Results in upward displacement of proximal fraagment and downward displacement of distal fragment - 4. Brachial Plexus: 1. Axillary n- dislocation of shoulder, abduction (deltoid) and lateral rotation (teres minor) are compromised. 2. Long thoracic n- winging of scapula (serratus anterior). 3. Radial n- wrist drop (extensors of forearm). 4. Median n- ape hand (thumb muscles) and flexors of forearm if damage is at elbow or above. 5. Ulnar n- claw hand and radial deviation of hand, loss of some flexors if at elbow or above. - 5. Peripheral nerves: 1. Common peroneal n- foot drop (tibialis anterior m) and inversion (peroneus muscles). 2. Deep peroneal n. entrapment- Compression of anterior compartment muscles of the lower leg by ski boot or athletic shoes that are too tight. Causes pain in the dorsum of the foot that radiates to the space between the first two toes.

www.usmle.tv

www.usmle.tv
- 6. Hands: 1. Carpel Tunnel Syndrome- compression of median nerve by inflammation, weakened flexion, abduction and opposition of thumb, loss of extension of index and middle fingers, sensory loss of index, middle and half of ring fingers and palmar part of thumb. 2. Cubital tunnel syndrome- sorry I was not able to find this one. 3. Dupuytren’s contracture- progressive fibrosis of palmar aponeurosis, pulls digits into marked flexion at MCP joints. - 7. Blood-testes barrier: There is a barrier that exists between the blood vessels that supply the testes (branches of the testicular artery and vein) and the duct system in which spermatozoa are produced and transported. The testis is derived partly from celomic mesoderm and partly from intermediate mesoderm with the blood vessels migrating in around the duct system. - 8. Abdominal arteries: 1. Celiac trunk(CT)-FOREGUT-left gastric a., splenic a., hepatic a. 2. Superior messenteric a.(SMA)- MIDGUT- part of duodenum through proximal 2/3 of transverse colon. 3. Inferior mesenteric a.(IMA)-HINDGUT- distal 1/3 of transverse colon to upper rectum - Collaterals: 1. Internal thoracic a. to superior epigastric a. to inferior epigastric a. 2. Superior pancreaticoduodenal a.(from CT) to inferior pancreaticoduodenal a. (from SMA) 3. Middle colic a. (from SMA) to left colic artery (from IMA) 4. Marginal a. (from SMA and IMA) 5. Superior rectal a. (from IMA) to middle rectal a. (from internal iliac a.) - 9. Bone: 1. Metaphysis: between epiphysis and diaphysis. 2. Epiphysis: growth plate responsible for linear bone growth. 3. Diaphysis: long part of bone responsible for annular bone growth.

NEUROANATOMY Ø INJURIES TO THE BRACHIAL PLEXUS: A REVIEW: [ Follow Ups ] [ Post Followup ] [ Step 1 ] Posted by Keri from IP 64.252.67.133 on March 06, 2003 at 05:46:40: Hi.. here are some useful notes... If anyone has more on this topic, please add. INJURIES TO THE BRACHIAL PLEXUS: 1. Waiter's Tip The waiter's tip injury occurs with damage to the superior trunk of the brachial plexus (C5-C6). The superior trunk can be damaged by stab wounds or when the head and trunk is separated from the shoulder during falls. The major nerves that would be damaged by this injury are the suprascapular, musculocutaneous, and axillary nerves. The major muscle affected by disruption of these nerves are the deltoid (abductor of the shoulder), supraspinatus (abductor of the shoulder), infraspinatus (lateral rotator of the shoulder), teres minor (lateral rotator of shoulder), biceps brachii (supinator and flexor of forearm), and brachialis (flexor of elbow). Injury to these muscles results in a position called the waiter's tip. As shown in the diagram below the limb hangs limply by the side, is medially rotated, with the forearm pronated due to loss of the supinating action of the biceps brachii. § Big Snell, pg 480 2. Claw Hand Claw Hand results from an injury to the inferior trunk of the brachial plexus (C8-T1). This injury results from excessive abduction of the arm such as when a person grasps onto something to prevent falling. The

www.usmle.tv

www.usmle.tv
nerve fibers from this trunk run in the median and ulnar nerve and supply all the small muscles of the hand (lumbricals and interossei). Paralysis of these intrinsic muscles of the hand causes the fingers to assume the "claw hand" position demonstrated below. This position is caused by the unopposed action of the extensor digitorum (which extends the metacarpophalangeal joints) and the flexor digitorum superficialis and profundus (which flex the fingers). Normally, these muscles are opposed by the lumbricals and the interosseus muscles. § Lecture Notes 3. Wrist Drop Wrist drop is caused by injury to the posterior cord and the radial nerve in the axilla. This injury can be caused by ill-fitting crutches or a downward dislocation of the humerus. Disruption of the radial nerve results in paralysis of the triceps, anconeus, and extensor muscles of the wrist. The person will be unable to extend the elbow, wrist, or digits. The resulting position of the upper limb, demonstrated in the diagram below, is called wrist drop. § Big Snell, pg 484 4. Median Nerve Palsy Injury to the median nerve within the elbow region results in some characteristic deficiencies (which you should be able to predict by understanding the distribution of the nerve). The arm muscles are not affected because none of them are supplied by the median. However, pronation of the forearm, flexion of the wrist and digits, and movement of the thumb are severely affected. The pronator muscles of the forearm and the long flexors of the wrist and fingers will be paralyzed, except for the flexor carpi ulnaris and medial half of the flexor digitorum profundus. When the patient tries to make a fist, as shown below, the index and to a lesser extent middle fingers remain straight, while the ring and little finger flex. § Big Snell, pg 486 5. Axillary nerve injury The axillary nerve may be injured by fracture of the humerus or dislocation of the shoulder. Following severance of the axillary nerve, the deltoid muscle is paralyzed and atrophies. 6. Ulnar nerve palsy The ulnar nerve can be damaged when the medial epicondyle of the humerus is damaged (like when you hit your funny bone). Ulnar nerve damage leads to paralysis of the flexor carpi ulnaris, medial half of the flexor digitorum profundus, and all of the small muscles of the hand except for the thenar muscles and the first two lumbricals. With paralysis of these muscles, a person is unable to flex the ring or little finger, adduct or abduct the digits, or adduct the thumb. The hand assumes the characteristic position shown below. The metacarpophalangeal joints become hyperextended due to paralysis of the lumbricals and interosseus muscles, which usually flex these joints. The interphalangeal joints are flexed- also due to paralysis of the lumbrical and interosseus muscles , which usually extend the joints. This condition is most marked in the medial two digits. In addition, there will be wasting of the hypothenar eminence and hollowing between the metacarpal bones due to atrophy of the hypothenar and interossei muscles. § Big Snell, pg 488

www.usmle.tv

www.usmle.tv
6. Winged Scapula Winged scapula results from injury to the long thoracic nerve, a supraclavicular branch (C5-C7) of the brachial plexus. This nerve to the serratus anterior lies on the medial wall of the axilla. The nerve may be injured by a stab wound, weight lifting, or a mastectomy. The paralyzed serratus anterior can no longer keep the scapula against the chest wall, which leads to the winged position shown below. § Big Snell, pg 481 KNOW THESE!! and also know how they would look like!!! ADDENDUM: Erb palsy may result in phrenic nerve damage(C3-5). Klumpke's palsy can cause Horner's syndrome, becuase the sympathetic innervation of the face arises from the T1 root. Know the dysfunction, too! Dysfunction of the long thoracic nerve leads to paralysis of the serratus anterior muscle. Affected patients may not complain of pain, but there is impairment in the last 30 degrees of overhead arm extension. The scapular rhythm also is disrupted, and the scapula may draw away from the thoracic cage; this is often referred to as winging of the scapula [2]. To demonstrate winging, the patient presses the outstretched arms against a wall; the involved scapula projects from the thorax as viewed from behind the patient (UpToDate) Ø > "Romberg sign"? - 1. ask patient to stand with feet together and CLOSE eyes- if sway-->POSITIVE ROMBERG! LESION IN DORSAL COLUMNS - 2. If patient sways with eyes OPEN ---> CEREBELLAR DAMAGE.. - Is it because when you close your eyes, you are totally relying on proprioceptive info going through the cerebellum where as with the eyes open, you are using visual clues to maintain balance Ø 1. Nerve involed when ptosis + dilated pupil? Also 2 . how will the eye look in Horners syndrome? - CN III palsy can cause ptosis, down and out eye deviation, and sometimes a dilated pupil and reduced accommodation. - Horner syndrome involves the triad of ptosis, miosis, and anhidrosis (lack of sweating due to a lesion of the cervical sympathetic nerve). - Ptosis with dilated pupil is due to CN 3. it innervates lavater pal sup, also CN 3 innervates, sphincter pupillea (NEAR RESPONSE)and ciliary - Also, Lesion of OPTIC NERVE: -If you place light in the normal eye--->Both eyes constrict. -If you place light in the affected eye --->Affected eye dilates. - Lesion of OCULOMOTOR NERVE: -If you place light in either eye--->Normal eye constrices and affected eye does not.

Ø Re: Pseudobulbar palsy - Pseudobulbar palsy results from the degeneration of corticobulbar pathways to V, VII, X, XI and XII cranial nerve nuclei with sparing of the III, IV and VI nerve nuclei. - Pseudobulbar palsy is a set of clinical signs on examination, not a diagnosis. The features include slowed slurred speech, difficulty with swallowing, weakness of face, tongue, and swallowing muscles, a

www.usmle.tv

www.usmle.tv
tendency for uncontrollable laughter or crying, and brisk jaw and gag reflexes. - The most common causes of pseudobulbar palsy are multiple bilateral strokes, multiple sclerosis, amyotrophic lateral sclerosis, progressive supranuclear palsy, some forms of static and progressive childhood diseases. Ø IMAGING TIPS (HY): TO BE CONFIDENT IN NEURORADIOLOGY , JUST GO TO YOUR MED SCHOOL/HOSPITAL LIBRARY AND PICK UP ANY 3 BOOKS IN RADIOLOGY WHICH HAVE CT SCANS/MRI. AT AN INTERN LEVEL THEY DO NOT EXPECT YOU TO DIAGNOSE RARE CONDITIONS , YOU SHOULD KNOW THE COMMON CT/MRI 1)CT HEAD-- HAEMORRAGE --SEEN AS WHITE OPACITY 2)SUBDURAL --SEEN AS CONVEX OPACITY CAN BE OF SAME ATTENUATION ASTHAT OF BRAIN BUT SHOULD BE IDENTIFIABLE 3)EXTRADURAL--- CONVEX OPACITY 4)MENINGIOMA-- DURA BASED BROAD MASS SMOOTH CONTOUR 5)GLIOBLASTOMA--- IRREGULAR MASS WITH ENTENSIVE SURROUNDING EDEMA 6)RING ENHANCING LESIONS--TUMORS/ ABSCESS/IN AIDS PATIENT IT IS TOXOPLASMOSIS 7)SUBARCHNOID---GOOD HISTORY PLUS BLOOD IN CISTERN THAT IS PROBABLY THE LIST FOR CT HEAD. MOST LIKELY ASKED. MRI 1) CORONAL VIEW-- PITUITARY TUMOR 2)ARNOLD CHIARI SYNDROME 3)SYRINGOBULBIA/MYELIA---DILATED CENTRAL CANAL 4)SPINAL CORD COMPRESSION THESE ARE THE MOST COMMON ONES START LOOKING UP IN THE FIRST BOOK FOR THESE SCANS AND UNDERSTAND THEM. NOW LOOK INTO THE SECOND BOOK AND TRY TO DIAGNOSE THEM SEE WHAT MISTAKES YOU MADE AND THEN GO THRU THE THIRD BOOK AND GIVE THE DIAGNOSE . I CAN ALMOST BET THAT YOU WILL SCOREMOST OF THEM CORRECTLY. ASHDOC TO BE CONFIDENT IN NEURORADIOLOGY , JUST GO TO YOUR MED SCHOOL/HOSPITAL LIBRARY AND PICK UP ANY 3 BOOKS IN RADIOLOGY WHICH HAVE CT SCANS/MRI. AT AN INTERN LEVEL THEY DO NOT EXPECT YOU TO DIAGNOSE RARE CONDITIONS , YOU SHOULD KNOW THE COMMON CT/MRI 1)CT HEAD-- HAEMORRAGE --SEEN AS WHITE OPACITY 2)SUBDURAL --SEEN AS CONVEX OPACITY CAN BE OF SAME ATTENUATION ASTHAT OF BRAIN BUT SHOULD BE IDENTIFIABLE 3)EXTRADURAL--- CONVEX OPACITY 4)MENINGIOMA-- DURA BASED BROAD MASS SMOOTH CONTOUR 5)GLIOBLASTOMA--- IRREGULAR MASS WITH ENTENSIVE SURROUNDING EDEMA 6)RING ENHANCING LESIONS--TUMORS/ ABSCESS/IN AIDS PATIENT IT IS TOXOPLASMOSIS 7)SUBARCHNOID---GOOD HISTORY PLUS BLOOD IN CISTERN THAT IS PROBABLY THE LIST FOR CT HEAD. MOST LIKELY ASKED. MRI 1) CORONAL VIEW-- PITUITARY TUMOR 2)ARNOLD CHIARI SYNDROME 3)SYRINGOBULBIA/MYELIA---DILATED CENTRAL CANAL 4)SPINAL CORD COMPRESSION THESE ARE THE MOST COMMON ONES START LOOKING UP IN THE FIRST BOOK FOR THESE SCANS AND UNDERSTAND THEM. NOW LOOK INTO THE SECOND BOOK AND TRY TO DIAGNOSE THEM SEE WHAT MISTAKES YOU MADE AND THEN GO THRU THE THIRD BOOK AND GIVE THE DIAGNOSE . I CAN ALMOST BET THAT YOU WILL SCOREMOST OF THEM CORRECTLY.

www.usmle.tv

www.usmle.tv
ASHDOC

Ø Re: Watershed Areas - Brain - GIT – Splenic flexure and Rectosigmoid - watershed areas are areas where there may be the first sign of any vascular insult.in the brain the region supplied by the ACA and the MCA and likewise in the GIT the area of the colon supplied by the SMA and the IMA...... - By disulfiramlike: - Watershed areas are those regions of the cerebral cortex that are at junctions of major arterial supplies. For example, consider the anterior cerebral artery and middle cerebral artery. The anterior cerebral artery supplies the region of the primary motor cortex (i.e. precentral gyrus)with cell bodies of upper motor neurons destined to eventually synapse with lower motor neurons that serve the lower extremities. The middle cerebral artery supplies the region of the primary motor cortex (i.e. precentral gyrus) with cell bodies of upper motor neurons destined to eventually synpase with lower motor neurons that serve the face and upper extremities. BUT - the region of the precentral gyrus containing the cell bodies of upper motor neurons that will synapse with lower motor neurons destined for the upper extremtieis lies BETWEEN THE TERRITORY OF DISTRIBUTION OF THE ANTERIOR AND MIDDLE CEREBRAL ARTERIES (i.e. between the medial part of the precentral gyrus and lateral part of the precentral gyrus). IT IS A WATERSHED AREA. BECAUSE IT IS POSITIONED FURTHEST AWAY FROM AN ARTERIAL SUPPLY, it is particularly susceptible to ischemia. A WATERSHED INFARCT produces an infarction in a watershed area. A WATERSHED INFARCT may arise from global cerebral ischemia secondary to hypoperfusion of the cerebral cortex (e.g. from heart failure). BECAUSE THE WATERSHED AREAS ARE FURTHEST FROM ARTERIAL SUPPLIES, THEY ARE PARTICULARLY SUSCEPTIBLE TO UNDERGO INFARCTION. The other clinically significant watershed area is the region of the cortex that lies in between the territories of distribution of the middle cerebral and posterior cerebral arteries. But, from my reading, the watershed area lying at the junction of the territories of distribution of the middle cerebral and anterior cerebral arteries is of most clinical significance. SUCH A WATERSHED INFARCT WOULD PRODUCE HAND WEAKNESS. Ø Spinal Cord Lesions: Amyotrophic lateral sclerosis:combination of UMN & LMN symptoms (pyramidal/anterior horn cells) Tabes dorsalis: bilateral posterior columns B12 def: post column & corticospinal Polio: anterior horn cells Guillian-barre: peripheral nerve invlt: motor and sensory Syringomyelia: anterior white commissure:bil pain and temp loss in tht area only ASA occlusion : all areas except the post cloumns PSA occlusion : ipsilateral post column Ø NERVE INJURIES: LOWER LIMB:hey the folln are effects of injuries to the various nerves(excluding any overlaps) 1)Superior gluteal nerve : supplies gluteus medius and minimus

www.usmle.tv

www.usmle.tv
weakness in abduction of thigh ,inability to stabilize pelvis (trendelenberg's gait) 2)Inferior gluteal nerve : supplies Gluteus maximus weakness in extension of thigh difficulty getting up from sitting position 3)Femoral nerve supplies anterior thigh muscles weakness or loss of flexion of hip & extension of knee(ANASTHESIA OVER ANTERIOR THIGH) 4)Obturator nerve supplies medial thigh muscles weakness or loss of adduction of thigh(ANASTHESIA OVER MEDIAL THIGH) 5)Sciatic nerve supplies posterior thigh muscles loss of function below knee weakness or loss of extension of thigh & flexion of knees(ANASHTESIA OVER POSTERIOR THIGH) 6).Tibial nerve supplies posterior leg muscles loss of inversion of foot,loss of plantar flexion of foot and flexion of toes (ALSO REMEMBER ANASTHESIA OF PLANTAR ASP OF FOOT& POSTERIOR LEG) 7)Common peroneal nerve supplies muscles of anterolateral leg region Loss of dorsiflexion of foot(foot drop) loss of extension of toes and loss of eversion of foot(ANASTHESIA OVER DORSAL ASP OF FOOT & ANTEROLAT LEG) Ø Epidural hematomas are the result (in almost all circumstances) of skull fractures, with sharp edges of bone cutting into the dural arteries (mostly the middle meningeal artery) which lie on the outer (periosteal) surface of the dura. Ø We have to know this... they could give a question with obstruction and ask where the accumulation results... anyway, here is the flow of CSF-- a mnemonic: CSF LIVES IN THE CNS, FLOODING FORWARD (LIKE ME), SPLASHING/SWIMMING, AROUND. CSF = CHOROID PLEXUS (site of production) LIVES = LATERAL VENTRICLE IN = INTERVENTRICULAR FORAMEN OF MONROE THE = THIRD VENTRICLE CNS = CEREBRAL AQUEDUCT FLOODING = FOURTH VENTRICLE FORWARD(LIKE ME) = FORAMEN OF LUSHKA AND MAGENDIE SPLASHING/SWIMMING = SUBARACHNOID SPACE AROUND = ARACHNOID GRANULATIONS (site of resorption) Ø EYE SIGNS: I found this useful table from CLINICAL NEUROANATOMY, by Waxman. YOKE MUSCLE COMBINATIONS CARDINAL DIRECTION OF GAZE-------------------MUSCLE --EYE UP,RIGHT---RT SUP RECTUS & LEFT INF OBLIQUE --EYES RT--RT LATERAL RECTUS,LEFT MEDIAL RECTUS. --EYES DOWN, RT---RT INF RECTUS & LT SUP OBLIQUE. --EYES DOWN, LEFT--RT SUP OBLIQUE & LEFT INF RECTUS. --EYES LEFT-----RT MEDIAL RECTUS & LEFT LATERAL RECTUS. -EYES UP,LEFT--RT INF OBLIQUE & LEFT SUP RECTUS

www.usmle.tv

www.usmle.tv
--------------------------------------------------PARALYSIS OF INDIVIDUAL MUSCLE MUSCLE------NERVE---DEVIATION OF---DIPLOPIA---DIECTION OF IMAGE EYE BALL PRESENT WHEN LOOKING 1.MED RECTUS-3---OUTWARD(EXT SQUINT)--TOWARD NOSE-direction of image vertical. 2.sup rectus-3-downward/inward--upward/outwarddirection of image-oblique. 3.inf rectus-3-upward/inward--downward/outward-direction of imageoblique. 4.inf oblique- 3-downward/outward--upward/inward-direction of image-oblique. 5.sup oblique-4-upward/outward--downward/inward-direction of image-oblique. 6.lateral rectus--6--inward(interal squint)--toward temple. direction of image- vertical Ø what is extra axial brain hemorrhage and intra axial brain hemorrhage? - Ans. Extra-axial bleeding which is usually subdural or epidural bleeding. This is blood that is on the surface of the brain and not in the substance of the brain. This has a characteristic appearance as bright color on the surface of the brain. A subdural blood clot would be biconcave or crescentic in appearance. An epidural blood clot is characteristically biconvex or lenticular in appearance. - Intraaxial bleeding is a hemorrhage in the substance of the brain and it has no characteristic appearance except that it is in the brain matter and not on the surface of the brain. Ø Visual Field Defects - The probable location of lesions producing visual defects is a favorite USMLE topic (and is also well worth knowing if you have occasion to work up such a patient). Here is a list that may help you sort through these problems: Central scotoma ~ macula Ipsilateral blindness ~ optic nerve Bitemporal hemianopia ~ optic chiasm Homonymous hemianopia ~ optic tract Upper homonymous quadrantanopia ~ temporal optic radiations Lower homonymous quadrantanopia ~ parietal optic radiations Also, cortical lesions produce defects similar to those of the optic radiations, but may spare the macula. Ø Amyotrophic lateral sclerosis: combination of UMN & LMN symptoms (pyramidal/anterior hoen cells) Tabes dorsalis: bil posterior columns B12 def: post column & corticospinal Polio: anterior horn cells Guillian-barre: peripheral nerve invlt: motor and sensory Syringomyelia: anterior white commissure: bilateral pain and temp loss in the area only Ø ASA occlusion: all areas except the post cloumns PSA occlusion: ipsilateral post column

www.usmle.tv

www.usmle.tv
NERVE INJURIIES: LOWER LIMB: the follng are effects of injuries to the various nerves(excluding any overlaps) 1)Superior gluteal nerve: supplies gluteus medius and minimus weakness in abduction of thigh ,inability to stabilize pelvis (trendelenberg's gait) 2)Inferior gluteal nerve: supplies Gluteus maximus weakness in extension of thigh difficulty getting up from sitting position 3) Femoral nerve supplies anterior thigh muscles weakness or loss of flexion of hip & extension of knee (ANESTHESIA OVER ANTERIOR THIGH) 4) Obturator nerve supplies medial thigh muscles weakness or loss of adduction of thigh (ANESTHESIA OVER MEDIAL THIGH) 5) Sciatic nerve supplies posterior thigh muscles loss of function below knee weakness or loss of extension of thigh & flexion of knees(ANESHTESIA OVER POSTERIOR THIGH) 6) Tibial nerve supplies posterior leg muscles loss of inversion of foot,loss of plantar flexion of foot and flexion of toes (ALSO REMEMBER ANESTHESIA OF PLANTAR ASP OF FOOT& POSTERIOR LEG) 7) Common peroneal nerve supplies muscles of anterolateral leg region Loss of dorsiflexion of foot(foot drop) loss of extension of toes and loss of eversion of foot( ANESTHESIA OVER DORSAL ASP OF FOOT & ANTEROLAT LEG) Ø APHASIAS 1]Aphasia--- unable to repeat sentence Type -- Speech- Comprehension -Localization a] Expressive (Broca) -- Nonfluent- good - Lower posterior frontal b] Receptive (Wernicke)-- Fluent -poor- Posterior superior temporal c] Conduction-- Fluent -good- Usually parietal operculum d] Global -- Nonfluent- poor -Large perisylvian lesion 2]Aphasia, able to repeat sentence well Type- Speech -Comprehension -Localization a]Tanscortical motor -- Nonfluent- good- Anterior to Broca's area or supplementary speech area b]Transcortical sensory -- Fluent -poor- Surrounding Wernicke's area posteriorly c]transcortical mixed -- Nonfluent- poor- both of the above d]Anomic-- Fluent- good -Angular gyrus or second temporal gyrus The term applies to those whose major symptom is word retrieval difficulties in spontaneous speech and naming tasks. The spontaneous speech of anomic aphasics is fluent and grammatically correct, but is marked by word retrieval failures. The word retrieval failures generate unusual pauses, circumlocution ("talking around" missing words) and substitution of nonspecific words such as "thing" for missing words.

www.usmle.tv

www.usmle.tv

Ø A 55-year-old man complains of numbness in both legs and progressive inability to walk over the past 2 months. Physical examination is normal except for a decreased perception of light touch and pain in the lower extremities as well as bilateral leg weakness. There is no sensory level. Laboratory workup is remarkable for a hematocrit of 30 percent and elevated total protein. Serum protein electrophoresis reveals an M spike. The etiology of this patient's weakness is most likely A necrosis of central nervous system gray and white matter B inflammation of dorsal root ganglia C loss of cerebellar Purkinje cells D elaboration of tumor-associated protein that elicits an immune response that is cross-reactive with peripheral nerves E tumor-elaborated immunoglobulin that is reacting with myelin components Ans. looks like peripheral neurophaty, in this case could be as a result of primary amyloidosis (ligth chain) so i go for... E- tumor-elaborated immunoglobulin that is reacting with myelin components Ø Patient has meningioma located in the parasaggital region of the falx cerebri, what neurological deficit might this produce?...Ans. leg weakness or paralysis Ø A 10 year old girl exhibits neurological signs, performance in school degenerates. several months later she develops a seizure, ataxia, and focal neurological symptoms. She is eventually quadraparetic, spasctic and unresponsive. She dies in one year. What viral disease did this child have at one year of age?.. and what is this disorder called?...ans. measles SSPE Ø Where do myxopapillary ependymomas frequently occur? medulla pons cerebellum conus medullaris cerebral ventricles ans. Conus Medullaris Actually Filum terminale Ø Lesions of which nerves will produce imposibillity of heel/toe walking? Ans. impossible heal with common peroneal & Tibial N 4 the toe.. Ø BRS says that in lesions of spinal cord: -lesion of lateral corticospinal tract---> ipsilateral motor deficit -lesion of ventral corticosp tract---> contralateral deficit This is true only when the lesion is under pyramidal decussation in first case and above spinal decusation in second case right? At what level is decussation of ventral tract? Thanks. Ans. A lesion of the lateral corticospinal tract in the spinal cord below the decussation always results in an ipsilateral spastic weakness below the lesion. The ventral corticospinal tract crosses at the level of the cervical cord where its axons innervate lower motor neurons to neck muscles. An isolated lesion of this tract in the neck (rare) may result in a

www.usmle.tv

www.usmle.tv
spastic weakness of cervical muscles, but since this tract is not well understood, and rarely lesioned in isolation, it is not a prominent test feature on the USMLE. Ø Name the site of lesion in the folln:(i would suggest u guys to draw a cut section of brain stem or spinal cord and then go abt these questions even though u will be able to guess the lesions right away..will be good for the exam..) - 1) Pt experiances decresred proprioception in left upper extremity ,decreased pain-temp sensation on right side of body below neck ,decreased pain -temp on left half of face.. - 2) Cerebellar dysfunction with right sided ataxia, loss of pain-temp over right face and left body,hoarseness, diff swallowing,loss of taste on right, vertigo &nystagmus - 3) Left hemipalegia with inabilty of right eye to abduct - 4) Paralysis of right lower facial muscles and right upper extremity and inability to adduct left eye, left ptosis & dilatation of left pupil, tongue deviates to right - 5) Left sided headache,total paralysis of left side of face with vertigo and left sided hearing loss - 6) Nystagmus, bilateral internuclear ophthalmoplegia, central scotoma of right eye, weakness of right lower extremity with postive babinski, urinary incontinence, right ptosis and difficulty in adducting right eye - 7) Pt has no pupillary reaction at all to light shined on left side.there is reaction to light in both eyes when light is shined on right side - 8) Pupillary reaction to light only on right side..whether light is shined on left or right eye - 9) Patient has inability to move right eye past midline on attempted left conjugate deviation but convergence is preserved. - 10) Neither eye blinks on touching right cornea but both eyes blink on touching left cornea - 11 )Only left eye blinks on touching either right or left cornea - 12) Tremor of right arms and legs due to a red nucleus lesion.. Which red nucleus is affected right or left? Ans. 1)left lower medulla 2) right upper medulla 3) right pons 4 left mid-brain 5) left sided ponto cerebellar tumor 6) multiple sclerosis 7) left CN 2 8) left CN 3 9) right medial longitudinal fasiculus 10) right CN 5 11) right CN 7 12) left red nucleus Ø A 35 year old female is brought to the emergency department by her husband who says that she complained of severe headaches and then lost consciousness. She is unable to mover her arms and legs, but can blink her eyes. In which of the following structures is this womans lesion located? cervical spinal cord

www.usmle.tv

www.usmle.tv
medulla oblongata midbrain pons thalamus I was wondering about the question. The answer can be Pons (bilateral medial inferior pontine syndrome) but there will be loss of tactile sensation from the trunk & extremities (Dorsal Coloumn medial lamniscus pathway)&CN 6 2nd answer can be medulla - medial medullary syndrome caused by the basilar artery but there will ba again Dorsal coloumn medial lamniscus pathway damage & CN 12 According to myself this one can be the Kernohan notch caused by the increase in the intracerebral pressure causes the damage to the corticospinal & corticobulbar fibers against the tentorium cerebelli Please write the explanation if you have Thanks Ans. the patient sustained a hemorrhage that has destroyed her ventral pons, descending corticospinal and corticobulbar fibers are interrupted. The patient is unable to move the facial, pharyngeal and limb musculature, leaving her paralyzed and unable to speak. This is locked in syndrome, patient is aware of being locked within her body. Only vertical eye movements and elevation of the eyelids are possible, so patient is blinking or moving their eyes. Lesions of the medulla typically involve the medial or lateral aspect. lesions of the lateral medulla is known as the posterior inferior cerebellar artery syndrome, or wallenberg syndrome Ø Medial inferior pontine syndrome: occlusion of the paramedian branches of the basilar artery Symptoms: 1)Loss of the contralateral spastic hemiparesis (Corticospinal tract) 2)Medial Lamniscus Lesion results contralateral loss of tactile sensation from the trunk & extremities 3) Cn6 damage results ipsilateral lateral rectus muscle paralysis (The imporant & easy differentiation) Ø Carotid artery occlusion : there are 2 carotid arteries external & internal. I do not know which one r u talking about or about common carotid artery. Ø Pontine lesions : there are 3 lesions 1) Medial inferior pontine syndrome Lateral rectus muscle damage medial strabism inabilty to abduct the eye 2) AICA : Horner's syndrome 3) MLF : here eye will deviate to the side of lesion CN3 palsy causes medial rectus muscle damage results in the inability to adduct the eye. Ø On CSF: CSF from the lumbar region contains 15-45 mg/dl protein and 50-80 mg/dl glucose. Protein concentration in cisternal and ventricular CSF is lower. Normal CSF contains 0-5 mononuclear cells. The CSF pressure, measured at lumbar puncture (LP), is 100-150 mm of H2O with the patient lying on the side and 200-300 mm with the patient sitting up. Ø CSF Blood Barrier: It's tight junctions between epandimal cells.(but in blood brain barrier,astrocytes+tight junctions of endothelial cells).

www.usmle.tv

www.usmle.tv
Ø On Bacterial Meningitis: Table 2.Typical CSF findings in patients with bacterial verses nonbacterial (aseptic) meningitis CSF characteristic-- Bacterial meningitis------------------------------------------------------------------------------Opening pressure----- Elevated (>180 mm H2O ------------------------------------------------------------------------------White blood cell count-- Increased (often >1,000/mm3), neutroplipredominance ------------------------------------------------------------------------------Glucose level Decreased (<40 mg/dL) ------------------------------------------------------------------------------CSF-serum glucose ratio <0.3 ------------------------------------------------------------------------------Protein level Increased (often >100 mg/dL) ------------------------------------------------------------------------------Gram stain results Stainable organisms present in 50%-80% of untreated cases ------------------------------------------------------------------------------Bacterial culture results Positive ------------------------------------------------------------------------------CSF, cerebrospinal fluid. > On Aseptic Meningitis: CSF characteristic Nonbacterial meningitis ------------------------------------------------------------------------------Opening pressure Normal or slightly elevated ------------------------------------------------------------------------------White blood cell count Increased (10-2,000/mm3), lymphocyte predominance

www.usmle.tv

www.usmle.tv

------------------------------------------------------------------------------Glucose leve; Normal (>45 mg/dL) ------------------------------------------------------------------------------CSF-serum glucose ratio >0.6 ------------------------------------------------------------------------------Protein level Normal or increased ------------------------------------------------------------------------------Gram stain resultsNo stainable organisms Bacterial culture results Negative Ø Causes of Aseptic Meningitis: Viral infection, SLE, Drugs (Ibuprofen?) Ø ON FRONTAL LOBE TUMORS: tumors affecting the frontal convexity, often produce 1] progressive hemiparesis, 2]focal or generalized seizures, and 3]mental changes. CONVULSIVE seizures may precede other symptoms by months or years. APHASIA may accompany a tumor of the DOMINANT hemisphere. A tumor at the BASE of the frontal lobes (particularly meningioma of the olfactory groove) can produce IPSILATERAL ANOSMIA; aAtumor on the MEDIAL surface can cause URINARY URGENCY or incontinence. MENTAL CHANGES, especially inattention and apathy, and ataxic gait are common when the tumor spreads across the CORPUS CALLOSUM to both frontal lobes Ø ON PARIETAL LOBE TUMORS: - May produce generalized or SENSORY focal seizures. Cutaneous tactile, pain, and temperature senses are UNIMPAIRED, but STEREOGNOSIS and CORTICAL SENSORY modalities (eg, position sense, two-point discrimination) are IMPAIRED contralaterally. - Contralateral homonymous hemianopia, apraxia, and anosognosia (no recognition of bodily defects) may also be present. - DENIAL of illness is characteristic. - SPEECH disturbances, AGRAFIA, and FINGER agnosia may occur when the tumor involves the DOMINANT hemisphere. Ø ON TEMPORAL LOBE TUMORS: - particularly in the NONDOMINANT hemisphere, often produce FEW EARLY symptoms but may cause convulsive SEIZURES - A tumor deep in the temporal lobe may cause CONTRALATERAL HEMIANOPIA, COMPLEX PARTAIL seizures, or convulsive seizures preceded by an OLFATORY AURA or VISUAL HALLUCINATION of complex formed IMAGES.

www.usmle.tv

www.usmle.tv
- Tumors involving the surface of the DOMINANT temporal lobe produce MIXED EXPRESSIVE and RECEPTIVE AFASIA or dysphasia, chiefly ANOMIA Ø ON OCCIPITAL LOBE TUMORS: - usually cause a CONTRALTERAL QUADRANT defect or HEMIANOPIA in the visual field with functional SPARING of the MACULA. Seizures may occur, preceded by an AURA of FLASHING LIGHTS but NO formed images Ø ON SUBCORTICAL TUMORS: - commonly involve the INTERNAL CAPSULE and produce CONTRALATERAL HEMIPLEGIA. - They may invade any lobe of the hemisphere, producing corresponding symptoms. - THALAMIC invasion produces contralateral CUTANEOUS sensory impairment. - Invasion of the basal ganglia does not usually produce parkinsonian symptoms but occasionally produces athetosis, bizarre tremors, or dystonic postures. - Hypothalamic tumors may produce eating disorders or, in children, precocious puberty Ø BRAINSTEM TUMOR: - are usually gliomas (usually astrocytomas). Common symptoms, resulting from destruction of NUCLEAR masses, are unilateral or bilateral PARALYSIS of the 5th, 6th, 7th, and 10th CRANIAL nerves and PARALYSIS of LATERAL GAZE. - Damage to the MOTOR or SENSORY pathways causes hemiparesis, hemianesthesia, or cerebellar disturbances (eg, ataxia, nystagmus, intention tremor). - Intracranial pressure increases late and only when tumors obstruct the aqueduct of Sylvius. Ø POSTERIOR FOSSA TUMOR: - including tumors of the 4th ventricle and cerebellum (usually medulloblastomas, gliomas, ependymomas, or metastases), interfere with CSF circulation and cause symptoms of INCREASED ICpressure early. ATAXIC gait, INTENTION tremor, and other signs of cerebellar dysfunction follow Ø CEREBELLOPONTINE TUMOR: - particularly NEURILEMMOMAS(acoustic neuromas, schwannomas), are characterized by TINNITUS, unilateral hearing impairment, and sometimes vertigo. - If the tumor is LARGE, pressure on ADJACENT cranial nerves, brain stem, and cerebellum produces loss of corneal reflex, facial palsy and anesthesia, palatal weakness, signs of cerebellar dysfunction, and, rarely, contralateral hemiplegia or hemianesthesia. - Loss of vestibular response to caloric stimulation, enlargement of the porus acusticus seen on imaging scans, and HIGH CSF protein content suggest acoustic neuroma Ø Nondominant parietal lobe lesions: - contralateral astereognosis&sensory neglect, anosognosia,construction apraxia,dressing apraxia,contralateral hemianopia or lower quadrantanopia Ø What tracts involved in Friedriech ataxia? What tracts in Vit B12 difficiency? Ans. the same tracts are involved in these 2 diseases: Vit B12 diff and Friedriech ataxia (AR disease: progressive ataxia, associated with pes cavus, DM, kyphoscoliosis, cardiomyopathy) dorsal columns,lateral corticospinal tracts and spinocerebellar tracts.

www.usmle.tv

www.usmle.tv
Ø damage to this nerve leads to loss of forearm pronation? Ans. Median Ø the nerve of adductor pollicis? Ans. ulnar nerve Ø 1) A 23 year old man with 10 year h/s of complex partial seizures and pinkish macular rashes on his face..wht is the diagnosis..wht are these rashes..wht is the inheritance pattern of the d/s? 2) A 47 year old had recently completed a course of chemotherapy for his acute myeloid leukaemia some 6 days previously. The nurse in charge asks you to examine the patient as he has noted the appearance of a widespread petechiae. He is afebrile and his clotting is normal. wht wud be the immed investigation and treatment..wht is the cause of these petechiae? 3) A 40 year old house-wife was referred to the local neurologist for investigation of dizzy spells and several episodes of loss of consciousness. According to her husband, she had been well until 4 months before, since then he had noted 3 occasions when he found it difficult to wake her up in the mornings. The GP was called in, but by the time he arrived the patient was awake and complaining of a headache. No obvious cause was found for these episodes. 1 month previously, after drinking 2 Gin and Tonics, she became unusually drowsy- this lasted for about 15 minutes. Following this, it was noted that she became talkative, but was speaking nonsense for a further 5 minutes. Three weeks before admission she had an epileptic seizure whilst at her daughter's wedding. There was no past medical history of note, although she had gained 5 kg in weight in the last 3 months. General medical examination was unremarkable. Pulse was 75/min SR, bp 130/85. Apart from her brisk tendon reflexes, neurological examination was likewise unremarkable, with flexor plantars. wht is the likely diagnosis?wht are the investigation? Ans. ???? Ø ions have higher levels in CSF when compared to plasma? Ans. compared to serum,CSF has the same concentration of Na+, higher Mg++,Cl and lower K+,Ca++,HCO3,Glucose,Protein Ø the location of Locus coeruleus?and related neurotransmitter? Ans. just lateral to the floor of 4th ventricle,yes NE Ø CNS structures without BBB(Blood Brain Barrier)? Ans. choroid plexus and also median eminence,neurohypophysis,lamina terminalis,pineal gland. Ø the presentation of conduction aphasia? Ans. poor repetition, good comprehension, fluent speech…yes!paraphasias(use of incorrect words),poor object naming,severe deficit of repetition and fluent speech is often indirect(circumlocutory). Ø Name the tissue that does not utilize glucose as the primary fuel source. What does it utilize then? Ans. cardiac beta oxidation of fatty acids.... Ø Wernicke’s aphaia sometimes misdiagnosed as psychosis,why? Ans. bcoz they usually have no hemiparesis,no depression,instead euphoric,absence of neurologic signs plus bizarre speech and ab behavior,agitation,paranoid thoughts that sometimes seen, lead to incorrect diagnosis of functional psychosis Ø WHERE IS THE CHEMORECEPTOR TRIGGER ZONE LOCATED? PONS 3RD VENTRICULE 4TH VENTRICULE FRONTALE LOBE MEDULA ans. rostral ventrolateral medulla... Ø

www.usmle.tv

www.usmle.tv

HISTOLOGY Ø Call-Exner bodies:Granulosa/theca cell tumor of ovary Councilman bodies:Hepatitis:toxic or viral Cowdry type A bodies:HSV,SSPE Russel bodies:Multiple myeloma Mallory's bodies:Chronic alchoholic hepatitis Schiller-Duval bodies:Yolk Sac tumor Ø SOME HIGH YIELD NOTES FOR CELL BIOLOGY & HISTOLOGY, CELL AND TISSUE BIOLOGY INTRODUCTORY STUFF DYES: Dye Structure: Chromophore Group: The chemical moiety of the dye that is responsible for its color. Auxochrome Group: The moiety on the dye that binds to the cellular components. It is usually either amino or SO42- groups. Amino auxochrome group = a basic dye. Sulfate auxochrome group = an acidic dye. Common types of stains: Hematoxylin and Eosin (H&E): Most common type of stain. Hematoxylin: Functionally a basic dye (despite the fact that it is anionic). It binds to basophilic (negatively charged) nuclear components like DNA and RNA. It stains blue Eosin: Acidic dye. It binds to positively charged, acidophilic components. It stains pink to red. Masson (Trichrome) Stain: Collagen is green. Elastic fibers are red. ACIDOPHILIC: Attracted to acidic substances, which are anionic (negatively charged) at physiologic pH. Thus acidophilic substances are positively charged. Proteins are acidophilic in at a pH higher (more basic) than their isoelectric point. When the environmental pH is above a protein's isoelectric point, the protein is positively charged and hence acidophilic. Many proteins are acidophilic at physiologic pH. Acidophilic Components: BASOPHILIC: Attracted to basic substances, which are cationic (positively charged) at physiologic pH. Thus basophilic substances are negatively charged. Proteins are basophilic at a pH lower (more acidic) than their isoelectric point. When the environmental pH is below a protein's isoelectric point, the protein is negatively charged and hence basophilic. Basophilic Components:

www.usmle.tv

www.usmle.tv
DNA and RNA = basophilic due to presence of phosphate groups. Proteoglycans = basophilic due to sugars and esterified sulfates which are negative at physiologic pH. Special Types of Staining Techniques: Metachromasia: A substance can take on a different than expected color when the substance has two chemically reactive groups that interact due to their close proximity. Fat-Staining: To stain membranes and lipid-materials, you must use a fat-insoluble solvent and freeze-fracturing. You can't use paraffin because it would dissolve the substance! Common solvents include propylene glycol, and ethanol. Sudan IV is a typical fat-soluble dye. The Schiff Reagent -- specific for DNA and polysaccharides. Feulgen Reaction: This reaction uses Leucofuchsin as a dye, which selectively stains purines in DNA. Periodic Acid-Schiff (PAS) Reaction: Selectively stains polyhexoses and hexosamines. Tissues stained by this reaction include: Glycogen Epithelial mucins in goblet cells. Proteoglycans in basement membranes -- but not of the CT matrix. Enzymatic Staining: For example, you can visualize mitochondria by testing for the product of a mitochondrial enzyme. The important point is that the enzyme is not stained directly in these procedures. Rather, the localization of its activity is tested for. Immunohistochemistry: Fluorescent Antibody Technique: Complex a fluorescent dye with an antibody that binds to specific antigens on tissues that you want to visualize. Indirect Immunofluorescence: Visualization of a tissue using two antibodies, where the target structure that is actually visualized is bound to the second antibody. Indirect Immunocytochemistry: Similar to indirect immunofluorescence, but eliminating the need for fluorescent visualization. Protein-A Gold Technique: Autoradiography: beta-electrons interacting with silver bromide (AgBr) crystals from radioactive materials illuminates radioactive structures. Electron Microscopy: Staining is usually with osmium. Some sort of fixation is required -- such as Freeze Fracture, in which we cut a preparation into thin slices using a microtome. PLASMA MEMBRANE AND BASIC CELLULAR STRUCTURES FLUID MOSAIC MODEL: RED-BLOOD CELLS GHOSTS: Put a RBC in salt and crack the membrane (i.e. make it leaky) so that all contents leak out. Then reseal the membrane, and we are left with topography maps of the RBC-membrane, showing peripheral and integral membrane-proteins. Integral Proteins: Glycophorin: Has extensive saccharide groups on the exterior surface. It is a single-pass protein. Band-III: Peripheral anion channel, exchanging HCO3- out for Cl- in. It is a multi-pass integral membrane protein.

www.usmle.tv

www.usmle.tv
Band-III has no lateral mobility in the membrane -- it is hooked directly to the cytoskeleton via Ankyrin ------> Spectrin ------> Actin spokes. Rhodopsin: The "mother" of the 7-pass alpha-helical multi-pass transmembrane protein (of the adrenergic G-protein-bound receptor family). This is a general class of integral proteins and describes a lot of different proteins. Peripheral Proteins Ankyrin is connected to the inside periphery of the RBC membrane. Spectrin is hooked to membrane via Ankyrin. Spectrin forms a lattice network composed of alpha and beta dimers. It hooks onto Band-III in the membrane (via ankyrin) at one end, and onto Actin at the spokes of the RBC-cytoskeleton in the RBC interior. Band 4.1: Another peripheral protein that helps anchor spectrin and actin to the RBC membrane. Hereditary Spherocytosis: Hemolytic anemia caused by a failure for RBC's to form a biconcave disc and therefore inability to squeeze through capillaries. It can be caused by any of a number of genetic mutations in RBC cytoskeletal proteins. One form is caused by a mutation in Ankyrin which results in bad splicing. There is a 2.1 and 2.2 splice out of the same precursor mRNA. 2.1 splice: predominant in developing cells. 2.2 splice required in mature cells. The 2.2 splice disappears with the missplicing mutation, hence RBC's mature but they don't function when fully developed. At the same time, other ankyrin isoforms of the same RNA precursor are translated normally, but they are in other cell-types. GLYCOSYLATION: N-Linked Glycosylation Sugar hooks onto Asparagine Residue. Common Sugars attached are N-Acetylglucosamine (GluNAc), and Mannose Glycosylation occurs cotranslationally, in the Rough ER. PROCESS: Core Glycosylation event occurs initially. It involves the linkage of the core oligosaccharide. The core oligosaccharide is then associated to the lipid complex, dolichol phosphate. Then it is disassociated and linked to the protein in one step. O-Linked Glycosylation Sugars hook onto Serine or Threonine residues. Common sugars attached are N-Acetyl Neuraminic (Sialic) Acid and NAcetylgalactosamine. Glycosylation occurs posttranslationally, in the Golgi. Experiments to Demonstrate the Fluid-Mosaic Model: Lipids can move laterally and can wiggle their hydrophobic tails very rapidly, but they can't flip-flop without a special catalytic reaction (catalyzed by flippase). Heterokaryon Experiment: Showed the movement of membrane proteins within the plasma membrane of a human-mouse hybrid. Fluorescence Recovery After Photobleaching (FRAP): A way to show that lateral movement of membrane proteins occurs. You can determine a Diffusion Coefficient for Lateral Mobility. Some common coefficients: Phospholipids in membranes: 1 x 10-8 cm2/sec

www.usmle.tv

www.usmle.tv
Most highly mobile membrane protein (Rhodopsin): 5 x 10-9 cm2/sec You start with 100% fluorescence in membrane, then zap with bleach a little spot on the membrane, and the fluorescence goes way down to about zero. Then you can watch the fluorescence recover (back up to near 100%) as adjacent lipids and/or proteins diffuse to the bleached area. Restricted Mobility: The cytoskeleton in red blood cells restricts the mobility of many membrane proteins on the RBC membrane. Cytoskeletal Elements: Filament Type Size Composition Microfilaments 7-8 nm Actin monomers Intermediate Filaments 10 nm variable Microtubules 25 nm alpha and beta tubulin monomers Myosin (Thick) Filaments variable Myosin Microtubules: Made of dimers of alpha and beta tubulin. They will self-assemble (autopolymerize) under the right conditions. Polarity (+)-End: Tubulin monomers are, on average, being added to this end. New monomers are put on at a faster rate than they fall off. (-)-End: Tubulin monomers are, on average, being removed from this end. Monomers fall off at a faster rate than they are put on. Microtubule Organizing Center (MTOC): Often found around centrioles. Microtubules hook to centrioles by their (-)-ends. Tread milling Effect: If you label one monomer on a microtubule, it will appear as if it magically moves from the plus to the minus end. That's because we keep adding new monomers to the plus end, so it gets pushed further back in the chain, until finally it is all the way toward the minus end and it falls off the chain. Anti-Microtubule Drugs: Colchicine: Binds to tubulin monomers and thereby prevents assembly of microtubules, killing the cell. Taxol: Controversial new anti-cancer drug that works in the exact opposite way as traditional drugs. It stabilizes the microtubule filament so that it can't disassemble. The result is the same, however: microtubule dynamics are lost and the cell dies. CYTOSKELETAL MOTOR PROTEINS: ATPases that cleave ATP to cause movement. The microtubules / actin don't move themselves. Rather it is the interaction of the motor proteins with the tubules that causes movement. Myosin: Actin-binding protein. Dynein: (-)-End Oriented Microtubule binding protein. It moves along the microtubules from the (+) to the (-) end. It therefore facilitates retrograde axonal transport. Tail is the region that attaches to the microtubules. The Head is the ATPase region. Kinesin: (+)-End Oriented Microtubule binding protein. It moves along the microtubules from the (-) to the (+) end. It therefore facilitates anterograde axonal transport. Cilia/Flagella: The minus end is toward the tip, and the (+)-end is toward the basal body, toward the plasma membrane. INTERMEDIATE FILAMENTS: Made of keratins, desmin, vimentin, and neurofilaments.

www.usmle.tv

www.usmle.tv
NUCLEAR TARGETING of PROTEINS: Nuclear Pores: Have specific targeting signals for nucleus-bound proteins. Pores are formed at points where the inner and outer layers of the Nuclear bi-membrane come together. EXPT: The Large-T Antigen of the SP40 virus was seen in the nucleus of a host cell by immunocytochemical imaging. A mutation on the T-Antigen site, exchanging a Lysine for a Threonine, caused sorting to occur in the cytosol instead. Thus this mutation was part of the Nuclear-Targeting Sequence. EXPT: Frog oocytes -- the results suggested that the nuclear targeting sequence was on the tail subunit of the nucleoplasmin protein in frog oocytes. When the head and tail were dissociated, the tail was able to through nuclear membrane and head wasn't. Also, if colloidal gold particles are associated with this tail subunit, they, too, can get into the nucleus, but only if ATP is present. SUMMARY: Transport into the nucleus does not take place by passive diffusion. It takes by highly specific transport with targeting sequences. It appears that nuclear transport is an active process (at least in frog oocytes). It requires ATP. ROUGH ENDOPLASMIC RETICULUM: Cytosolic proteins can be synthesized on free ribosomes instead of the Rough ER, per se. However, the following proteins are always synthesized on the Rough ER: Membrane Proteins: Using Signal Peptides and Signal Recognition Particles, they are directly translated into the membrane, where they stay. Secreted Proteins: They are exuded into the ER lumen, and then onto Golgi and finally secreted in vesicles. They must be synthesized on ER therefore. MITOCHONDRIA: Proteins destined for the mitochondria are integrated into the mitochondrial membrane post-translationally. First they are synthesized, and then they go to mitochondria via a vesicle. GOLGI COMPLEX: Cis Golgi: Earliest part of Golgi, closest to the ER. Transition Vesicles often transport material from the ER to the Golgi. Middle Golgi Trans Golgi: Part of Golgi off of which vesicle bud. ENDOCYTOSIS: Clathrin associated with a receptor protein, which in turn associated with the membrane. There are several adapter proteins, depending on the membrane to which the vesicle will fuse. For example, there is a specific adapter protein for the Golgi. The difference in adapter proteins between LYSOSOMAL STORAGE DISEASES: Lots of diseases have at least one etiology where the mutation lies in incorrect sorting of the protein, rather than a non-functional protein itself. I-Cell Disease:

www.usmle.tv

www.usmle.tv
The Mannose-6-Phosphate recognition marker is found on one of the NLinked Oligosaccharides of a lysosomal hydrolase. It targets the protein for the lysosome. Adding the M6P is a two step process. One enzyme puts on N-Acetylglucosamine phosphate onto a mannose residue. A second enzyme then removes the N-Acetylglucosamine, leaving Mannose6-Phosphate in its wake. It is the first step, addition of N-Acetylglucosamine phosphate, that goes wrong in ICell disease. Cystic Fibrosis: The CFTR protein is mostly getting made, but it is not getting transported to the Golgi. The primary etiology of the disease is a sorting problem, not a defective protein. Tay-Sach's Disease: Again, one of the causes is a missorting of the protein betaHexosaminidase, where it can't get from ER to Golgi. Emphysema and Familial Hypercholesterolemia are two more examples. Sucrase-Isomaltase Deficiency: The Sucrase-Isomaltase enzyme is normally targeted to the apical epithelial membrane and is involved with disaccharide / glycogen breakdown. Individuals with the defect can't metabolize long-chain sugars. Again it seems that the secretory pathway for the enzyme is blocked. EPITHELIA EPITHELIAL CELL TYPES: Simple Squamous Epithelium: Kidney Bowman's Capsule Resemble fried eggs in shape. Simple Cuboidal Epithelium: Kidney Collecting Tubule Kidney tubules cells are specialized for absorbing salt and water in an apical to basal direction. Simple Columnar Epithelium: GI Tract (Stomach, Jejunum, Duodenum, Ileum) Other Tissues: Gall Bladder and Uterine Gland. Simple Columnar Cells are specialized for one or all of three things: Secretion Protection Absorption: This is especially true in Duodenum and Jejunum. They have oval nuclei toward the basal side. SIMPLE COLUMNAR EPITHELIUM CELL TYPES: There are four basic cell types of simple columnar epithelia Columnar Fusiform Basal Goblet: = Modified columnar cells that synthesize and secrete mucous. Stereocilia are "cilia" that don't move, but they are actually very long microvilli specialized for absorption, and only visible at EM level. Pseudostratified Columnar Epithelium: Trachea and Upper Respiratory Tract The trachea is actually ciliated, but there are also non-ciliated pseudostratified columnar epithelia.

www.usmle.tv

www.usmle.tv
Example of Pseudostratified Non-Ciliated Columnar Epithelium: Male Urethra Stratified Squamous Epithelium: Salivary Glands, Skin, Vaginal Wall There was no example of this in the carousels but only final testing slide. Stratified Squamous Keratinized: Layer of Keratin on top, as in Skin. Stratified Squamous Non-Keratinized (Mucosal): No Keratin on apical surface, as in Vagina and Mouth. Stratified cells form the following layers: Basal End: Cuboidal Cells that are proliferative. Middle: Polygonal cells held together by desmosomes. Apical End: Squamous Cells that are non-proliferative. Stratified Cuboidal Epithelium: Sweat Duct of Skin Transitional Epithelium: Urinary Bladder The tissue appears to transform from 5-8 layers when empty, to 2-4 layers when the bladder is filled. The cells can squish together. EPITHELIAL General Characteristics AVASCULAR: Epithelial Tissue is generally avascular. POLARITY: Epithelial cell have polarity. The apical side often contains microvilli and faces the lumen of whatever surface the epithelium lines. Microvilli are characteristically found on apical domain. Actin filaments are associated with the microvilli, forming the terminal web. Cilia are found on apical membrane, in ciliated cells. The basal side is opposite that. A basement membrane, consisting of a basal lamina and reticular lamina, often underlies that. The Na+/K+-ATPase pump is characteristically only found on the basolateral membrane. BASEMENT MEMBRANE: The basal lamina is visible only at the EM level. The Basement Membrane, on the basal surface, is available at the LM level and consists of the basal lamina plus the underlying connective tissue. MESOTHELIUM: Mesodermally derived epithelium that lines body cavities. TERMINAL WEB: Visible network of actin filament on the apical end of an epithelial cell. JUNCTIONAL COMPLEX: The junctional complex keeps the apical and basal sides of the epithelium separate from each other. Zonula Occludens: Tight Junctions. They allow for selective passage of particles, and they prevent particles from getting stuck between cells or getting into the lumen. Zonula Adherens: Also present at the junctional complex. Macula Adherens: Desmosome. It goes all the way around the circumference of the cell, like a belt or a spotweld. TERMINAL BAR: Zonula Occludens + Zonula Adherens. Gap Junction: Believed to mediate electronic coupling between cells. Dye can squeeze through a gap junction to get one from cell to the neighbor. POLARITY EXPT: Cells lost their polarity by disassociating and then reassociating cells such that they lose their intercellular contacts. The Na/K ATPase pump occurs only on the basal membrane of the cell. Viral EXPTs: You can also study the distribution of viral proteins to study the host-cell's machinery, since the virus uses the host-cell's machinery.

www.usmle.tv

www.usmle.tv
People have watched where viral capsid proteins went when they associated with the host plasma membrane. The Influenza Virus only distributed proteins to the apical end of an epithelial cell. PATHWAYS for Explaining Polarity: Two alternative methods have been figured out. Targeting Mechanism where a class of vesicles specifically recognize proteins on the apical domain. Hence some proteins will only merge with membrane on the apical domain. Transcytosis: Some evidence also suggests that proteins are initially sorted in the basal domain, and then later transferred to the apical domain via transcytosis. EPITHELIAL EXOCRINE GLANDS: Unicellular: Goblet Cells are unicellular exocrine glands. Simple Tubular Simple Branched Tubular Simple Alveolar Simple Branched Alveolar Compound Tubular Compound Alveolar Compound Branched Tubular Compound Branched Alveolar THE CELL CYCLE Types of Cells Cycles: Chromosomal Cycle Centrosomal Cycle: The Centrioles duplicate themselves prior to mitosis, and move to opposite poles. Cytoplasmic Cycle: Refers to cytokinesis. Distribution and redistribution of cytoplasm. Phosphorylation Cycle: Phosphorylation promotes mitosis, as discussed later. Nuclear Membrane Cycle: Nuclear Lamins are phosphorylated during Prophase, which causes them to dissociate and results in breakdown the nuclear membrane. Nuclear lamins are a form of intermediate filament. Nuclear Lamins are dephosphorylated during telophase, so they reassociate and membrane reforms. CENTROSOMES: They divide into two before mitosis. They form the Microtubule Organizing Center, out of which the mitotic spindle grows, during mitosis. MITOSIS: Prophase: Nucleoli disappear Centrosomes split and each daughter forms an aster. Prometaphase The Nuclear Envelope breaks down. Microtubules from each centrosome start interacting with the chromosomes. Kinetochore Microtubules from the centromere of each chromosome mature and attach to some of the spindle microtubules. Metaphase The Kinetochore microtubules align the chromosomes along the metaphase plate.

www.usmle.tv

www.usmle.tv
The chromosomes are held in place by the opposed kinetochores and their associated microtubules. Anaphase Kinetochores on each chromosome separate, allowing each chromatid to be pulled toward the poles. Anaphase-A: Kinetochore Microtubules shorten. Since the plus end of these microtubules is right at the centromere, this shortening causes the chromosomes to be pulled toward the poles. Anaphase-B: Polar Microtubules elongate. The plus end of the polar microtubules face the equator too, but this elongation somehow aids in pulling (or pushing) the poles apart. Ca+2 seems to play a role in promoting anaphase. There is high Ca+2 concentration during anaphase. Telophase: Daughter chromatids reach the poles. Kinetochore microtubules disappear. Nuclear envelope reforms as nuclear lamins reassociate, condensed chromatin expands, and nucleoli reappear. Involves dephosphorylation of many proteins. Cytokinesis. Actin and Myosin pinch the cell and form a contractile ring. Organelles and cytoplasm are distributed evenly. KINETOCHORES: Protein masses that form at the centromeres during mitosis, and to which kinetochore microtubules attach. SCLERODERMA: These patients produce auto-antibodies that react specifically with kinetochores. The Kinetochore Microtubules elongate toward the chromosome! They have their plus-end facing the chromosome, hence they shorten during chromosome separation. Both Kinetochores must be attached for the separation to occur. This is a biological safeguard to assure that nondisjunction does not occur. CELL FUSION EXPERIMENTS: They provided evidence for activators that promoted mitosis and DNA Synthesis. Cells in different stages of the cell cycle were fused together to see what would happen. G1 Cell + S Cell: G1 Cell immediately goes into DNA-Synthesis. This is because the S-Cell had S-Phase Activator, which promoted theG1 cell to go into S-Phase. G1 Cell + G2 Cell: G1 will go through S-Phase as normal until it reaches G2, then the two cells will go through mitosis together. So, the G2 cell waits for the G1 cell to catch up with it. This suggests that S-Phase Activator present in the S-Phase is no longer functional in the G2 phase. This is important -- it prevents polyploidy by not allowing cells to synthesize DNA twice! G2 Cell + S Cell: Again, S-Phase cell catches up to G2 cell, then they proceed through mitosis together. This expt demonstrated that their was no S-Phase Inhibitor in the G2 cell, or else the S-cell wouldn't have completed mitosis. Thus there must be some other explanation for why the G2 cell doesn't undergo replication in presence of S-Phase Activator. Any Interphase Cell + M-Phase Cell: The interphase cell will prematurely enter mitosis, from any stage, resulting in an abnormal cell. This is mediated by M-Phase Promoting Factor (MPF), as below.

www.usmle.tv

www.usmle.tv
DNA-DAMAGE: When G2 cells are irradiated, their entry into M phase is delayed. They don't enter mitosis until their DNA-repair processes are complete! CELIAC DISEASE: Intestinal disease results from abnormalities in intestinal epithelial cell division. Cells normally divide at the crypt (basal) region of the cell -- not the apical end. For each dividing cell, one daughter will become an epithelial cell and migrate toward apical surface, while the other will remain a crypt cell. In Celiac Disease, this process does not occur normally. M-PHASE PROMOTING FACTOR (MPF): Xenopus Oocyte MPF Levels: Oocyte: MPF level is low, in order to freeze egg in prophase, and to prevent mitosis. Mature Newly Laid Egg: MPF Level is high Early Embryo: MPF levels alternatively high in M-Phase and low in Interphase. STRUCTURE: It has two subunits CYCLIN: The regulatory subunit. It is produced at a constant rate in the cytoplasm. CDC2: The kinase subunit. It phosphorylates targets to induce mitosis. CELL DIVISION CYCLE: Pre-MPF is an inactive form of Cyclin + CDC2 is sitting around in cytoplasm. Active-MPF is made by a combination of two things: Kinase Cascade from signal transducers modifies the Pre-MPF in complex reactions (multiple phosphorylations) to active MPF. Cyclin levels accumulate in the cytoplasm, as cyclin is continually made in many cell types. Mitosis is induced by Active MPF, via the catalytic activity of the cdc-2 subunit. Active MPF also produces cyclinases -- cyclin degradation enzymes that lower the levels of cyclin. This inactivates MPF, until cyclin is resynthesized or until it accumulates again in the cytoplasm MUSCLE SARCOMERE COMPONENTS: Z-Disk: The union of two actin heads. It demarcates the sarcomere. At the Z-Disk, there is no myosin. A-Band: The distance of one thick filament, consisting of two myosin filaments. I-Band: The distance from the end of one thick filament to the beginning of the next thick filament. During contraction, the I-Band becomes shorter. The I-Band consists entirely of actin. The I-Band marks the margins of two adjacent sarcomeres. Each I-Band technically lies within two sarcomeres. H-Zone: The distance from the end of one thin filament to the beginning of the next thin filament. During contraction, the H-Zone becomes shorter. The H-Zone consists entirely of myosin.

www.usmle.tv

www.usmle.tv
The H-Zone lies completely within the sarcomere, near the center of the sarcomere. ACTIN MYOSIN INTERACTION: In a myofibril, in cross section: Six actins can interact with each myosin. Actins are in a hexagonal array. Three Myosins can interact, in triangular fashion, with each actin. SKELETAL MUSCLE CONTRACTION: Myosin plays the role of an ATPase ActinBinding Motor Protein. We will start with myosin bound to actin. When Myosin is bound to Actin, ATP is bound to the myosin head. With ATP bound, Myosin can then detach from the actin thin filament. Once detached, the myosin is free to hydrolyze the bound ATP to ADP + Pi. It hydrolyzes the ATP, and the ADP + Pi remain attached to the myosin head. The myosin then reattaches to the thin filament. Reattachment leads to the release of the Pi group, which in turn strengthens the interaction between the actin and myosin. Power Stroke: With the ATP gone, the myosin head undergoes a conformational change, causing the actin and myosin to move relative to each other. Then the myosin head releases the ADP. Then Another ATP must bind to the myosin, in order for the myosin to release from the Actin to start another cross-bridge. If there is no more ATP, Rigor Mortis results, in which the muscle is stuck in the contractile state, with myosin bound to actin. REGULATION OF THE CROSS-BRIDGE CYCLE: Regulation is according to intracellular levels of Calcium and is mediated by Troponin Complex and Tropomyosin. RELAXED STATE: Tropomyosin is bound to the thin filament around its major groove, in the absence of calcium. The Troponin Complex is periodically bound to the thin filament such that it blocks the interaction between Actin and Myosin. CONTRACTED STATE Calcium binds to the Troponin Complex, causing a conformational change in Troponin-C. Troponin Complex (Troponin plus tropomyosin) removes itself from the thin filament as a result, such that Myosin can bind. ORGANIZATION OF MUSCLE: MUSCLE: A whole muscle is surrounded by an epimysium membrane. It is composed of a bundle of fasciculi. FASCICULUS: Each fasciculus is surrounded by a perimysium membrane. It is composed of a bundle of myofibers. MYOFIBER (MUSCLE FIBER): Each muscle fiber is surrounded by an endomysium membrane. It is composed of a bundle of myofibrils. It is a very long and thin single muscle cell. It has a sarcolemma plasma membrane, with an endomysium basement membrane beyond that.

www.usmle.tv

www.usmle.tv
MYOFIBRIL: A bundle of myofilaments, stacked neatly next to each other such that the Z-Disc is lined up. Every Thin filament in a myofibril can interact with 3 thick filaments. Every thick filament in a myofibril can interact with 6 thin filaments. Each Myofibril is bathed in sarcoplasm and surrounded by a sarcoplasmic reticulum from whence it gets it calcium supply. MYOFILAMENT: A very long, continuous series of sarcomeres, consisting of actin and myosin. Thin Filament: Actin Thick Filament: Myosin Intermediate Filament: Some muscle fibrils also have some intermediate filaments. SKELETAL MUSCLE CROSS-SECTION (Location of Nuclei): The nuclei are all pushed to the periphery, because the actin/myosin fibers take up the central part. Compare this to cardiac muscle, whose nuclei are in the center. CARDIAC -VS- SMOOTH MUSCLE: Cardiac muscle has nuclei centrally located and relatively more cytoplasm than smooth muscle. T-TUBULES: They run in the triad, with sarcoplasmic reticulum on either side, in between each of the individual myofibrils. They transmit the Ca+2 depolarization from the plasma membrane to the SR, which in turn transmits it to all the fibers. Ca+2 release from the SR initiates the muscle contraction. Ca+2 is pumped back into SR to restore resting, by a Ca+2-ATPase. NEUROMUSCULAR JUNCTION: Active Zone: Electron-dense (dark in EM scan) patch of membrane at the end of a nerve, right at the neuromuscular junction. Note that vesicles are found right at the membrane, while mitochondria are found more proximal, away from the active zone. Junctional Fold is right opposite the active zone. Ach Receptors on the muscle membrane are highly concentrated right at the nerve terminal.

MUSCLE DEVELOPMENT: Mesenchymal cells form myoblasts. Myoblasts proliferate and form myotubes by fusing together, resulting in a large multinucleate cell. So, muscle becomes multinucleated by the fusing together of primitive myoblasts. SATELLITE CELLS: These cells lie squeezed in-between the endomysium (basement membrane) of a myofibril and the fibers themselves. Developmentally they have the same origin as myotubes. They are myoblasts that did not fuse with other myoblasts during development. FUNCTION = Muscle Repair. They proliferate to repair damaged muscle tissue. They will divide to regenerate muscle, but the regeneration may be incomplete. MUSCLE REGENERATION:

www.usmle.tv

www.usmle.tv
When the muscle fibers are gone, all that is left is the basal lamina and reticular formation of the endomysium. The satellite cells then migrate into the empty endomysium. Macrophages come in to remove necrotic remnants (debris) Muscle regeneration may be incomplete (muscle atrophy or weakness). Fiber Splitting can occur, where the satellite cell can generate smaller duplicated myofibril sections from one original myofiber. DUCHENNE MUSCULAR DYSTROPHY: Poor function and structure of skeletal muscle. Symptoms / Prognosis: Hypertrophy of lateral thigh and calf, except that it is not muscle -it is fatty tissue. Death by respiratory failure, usually due to infection and or regurgitation. Esophagus malfunction: The skeletal muscle portion of the esophagu1s doesn't function right, leading to problems with swallowing and regurgitation. Upper third of esophagus: skeletal muscle Middle third of esophagus: Transition of half skeletal and half smooth muscle. Lower third of esophagus: Smooth muscle. Gower's Sign: Diagnostic test of ability to squat down and stand back up. Histopathology: You see necrotic muscle fibers, that ultimately fill with fat infiltrates, giving the pseudohypertrophic appearance to the muscle. Pathology: Faulty Dystrophin Gene, resulting focal lesions on the muscle membrane ------> Calcium leaks in the cell ------> perpetual contraction ------> necrosis You get contracted myofibers. You get swollen mitochondria. The fibers remaining (that are not necrotic) are spheroid. GENETICS: X-Linked recessive disorder. It is passed from Mother to Son (hemizygous) on the X-chromosome. DMD Gene, coding for Dystrophin, is very large. Many of the mutations are new mutations. There are brain and cardiac isoforms of the Dystrophin protein. Werdnig Hoffman Muscular Dystrophy: Variant wherein a small portion of the dystrophin is missing. In DMD, a large portion is missing. DYSTROPHIN: Function is to link the muscle fibers with the extracellular matrix. It function in a spectrin-like fashion, to connect the extracellular matrix with muscle actin. This provides muscle membrane stability. Beyond that function is unclear. TREATMENT METHODS: Satellite Cell Replacement They tried to inject donor satellites to provide donor dystrophin, but the dystrophin couldn't get past the basement membrane barrier to get to the membrane. Using collagenase for this purpose helped but didn't increase muscle strength. Viral Infection with the Correct Gene -- severe limitation here was the huge size of the DMD gene. Repair Point Mutations on mRNA -- Novel approach where they repair the mRNA to get past the stop codon point, suppling an artificial amino acid at that point.

www.usmle.tv

www.usmle.tv
In-Vitro Screening: Extract cells from the embryo and test for a particular exon on the DMD gene If the embryo had the DMD gene, then a positive PCR product would be obtained (i.e. some of the exons were not there). If the embryo did not have the DMD gene, then a negative PCR product was obtained, and they could reimplant the embryo for development. PENNIFORM MUSCLE: Muscles with a central tendon, used for strength and stability. Example = Transversus Abdominis. FUSIFORM MUSCLE: Muscles with a tendon on either side longitudinally, used for speed. Example = Biceps Brachii. Ways of Distinguishing CARDIAC MUSCLE -vs- Smooth Muscle: Cross-Section: Cardiac Muscle has a centrally placed nucleus, whereas the nucleus is around the periphery in skeletal muscle. Longitudinal Section: Cardiac muscle appears striated, but with branches. The cardiac cells are branched in longitudinal section. The cardiac cells have the same structural units as skeletal muscle, although SR and T-Tubules won't be as regular. In Cardiac Cells you get a diad instead of a triad -- one SR membrane will adhere with one T-Tubule. INTERCALATED DISK: The junctional complex that separates cardiac muscle cells. They always coincide with the Z-Line of muscle fibers. Fascia Adherens is the basic structural connections between the two cells. They are similar to desmosomes but are only found in cardiac cells. The Fascia Adherens apparently binds thin filaments in adjacent I-bands to the plasma membrane of cardiac cells. Desmosomes: The tightest point of connection between two cardiac cells. Gap Junctions: Allows fast electrical conduction between two cardiac cells. CARDIAC ISCHEMIA: Structural changes in ischemia: 15 minutes: Structure changes occur. 30-60 minutes: The cell can still recover. > 60 minutes: The cell dies, necrosis. Reperfusion Injury: Occurs when oxygen is suddenly replenished after extended deprivation. It can cause mitochondria to swell up and explode. Histopathology of Cardiac Ischemia: Chromatin is more condensed than normal. Mitochondria swell Glycogen stores are absent. Unlike skeletal muscle, cardiac muscle cannot regenerate. SMOOTH MUSCLE: Histological Characteristics Single central nucleus, but the amount of cytoplasm is less as compared to cardiac muscle, i.e. the nucleus takes up a great space in the cell in smooth muscle. Cell is not striated, as actin and myosin are not arranged in linear fashion.

www.usmle.tv

www.usmle.tv
The amount of actin is greater than that of myosin. Actin is bound to dense bodies in the cytoplasm, which are held in place by intermediate filaments. CONTRACTION: RELAXED STATE: Myosin thick filaments are sparse, i.e. they are not polymerized. Myosin is dephosphorylated when relaxed. CONTRACTED STATE: Myosin Light Chain is phosphorylated. Myosin forms more thick filaments This allows the dense bodies to move toward each other. PROCESS OF CONTRACTION / REGULATION Calcium activates Calmodulin Complex. Calmodulin Complex then activates the Myosin Light Chain Kinase (MLCK). Myosin Light Chain Kinase then phosphorylates the myosin light chains DOWN-REGULATION: Here are ways of inducing relaxation or lessening contractile tonicity. beta-Adrenergic transduction can phosphorylate the Light Chain Kinase, thus deactivating it ------> No Phosphorylation of Myosin Light chains ------> Less contraction. Phosphatases remove the phosphate from the myosin light chain to induce relaxation. ACTIN-BASED MOTILITY: Pseudopod Movement: Cytoplasmic streaming as mediated by actin polymerization and depolymerization. No myosin is involved. Cytokinesis: Once again involves interaction of actin and myosin to pinch the cell. MICROTUBULE BASED MOTILITY: Dynein and Kinesin Dynein is a minus-end protein. It travels from plus to minus and thus aids in retrograde axonal transport. Kinesin is a plus-end protein. It travels from minus to plus and thus aids in anterograde axonal transport. CONNECTIVE TISSUE COMPONENTS OF CONNECTIVE TISSUE: Fibers Collagens Elastic Fibers Ground Substance (Proteoglycans) Cells Macrophages Mast Cells Fibroblasts COLLAGEN: The primary fiber found in connective tissue. Although other elastic fibers are also found. Tropocollagen is the basic structural unit, consisting of three alphachains arranged in a helix. Tropocollagen shows a typical banding pattern on EM, due to the staggered helices. Procollagen doesn't show the banding pattern. Chemistry: Every third residue is glycine. Hydroxyproline and Hydroxylysine are also prevalent. Synthesis:

www.usmle.tv

www.usmle.tv
Registration Peptide: The registration peptide, distinct from the signal peptide, accomplishes two things. It keeps the collagen helix soluble in the cell. It allows the alpha-strands to align properly in the cell, in order to form the helix. alpha-strands are synthesized in the ER as usual. The signal peptide is cleaved but the registration peptide, as above, remains. Post-Translation Modifications: Lysyl Hydroxylase and Prolyl Hydroxylase hydroxylate lysine and proline residues. Various glycosylations are done. Procollagen is formed intracellularly. It is the soluble, spontaneously formed helix that results from the individual strands, after posttranslation modifications are made: Procollagen still has the registration peptides intact. Procollagen is secreted. Procollagen Peptidases then cleave the registration peptide extracellularly, to result in Tropocollagen. Tropocollagen then forms fibrils spontaneously, stabilized by crosslinks. Lysyl Oxidase turns on Hydro lysine residues into aldehydes, to stabilize cross-link formation. Fibers form by the association of fibrils. Collagen Types: Collagen I: Skin + Bone Collagen II: Cartilage Collagen III: Aorta (Reticular Fibers) These are also associated with elastic fibers A silver stain will only stain reticular fibers, so they can be identified. Collagen IV: Basement Membrane Basement membranes retain the registration peptide. As a result they don't form fibers but instead form sheets. COLLAGENASE: Breakdown of Collagen Process of Collagen Degradation: Collagenase is secreted as a proenzyme and is activated by other proteases. It cleaves at a specific site -- about 25% of the way down the molecule. The specific cleavage results in the spontaneous denaturation of the collagen helix. The smaller pieces have a lower melting point and are more volatile. Other proteases then finish off the job. Collagenase activity is temperature and fluid-dependant REGULATION of COLLAGENASE: Tissue Inhibitors of Metalloproteases (TIMPs): They bind only to activated collagenases, thus moderating their activity through negative feedback. Extracellular Proteases: Three types of extracellular proteases aid in the degradation of collagen: Metalloproteases. Collagenase is a metalloprotease Serine Proteases. For example -- elastase and thrombin Cathepsins. Collagen-related disorders Ehlers-Danlos Syndromes: Hyperextensibility of skin and joints.

www.usmle.tv

www.usmle.tv
Osteogenesis Imperfecta Recessive Dystrophic Epidermolysis Bullosa: Too much collagenase. Scurvy: Vitamin-C deficiency leads to malfunctioning prolyl hydroxylase. ELASTIC FIBERS: Arrangements of elastic fibers: They can be arranged in three different ways Fibers / Fiber Bundles -- as in skin Lamellae (sheets) -- as in vasculature Fine Networks -- as in the lung Protein Composition: Microfibrillar Protein: Forms the underlying "scaffolding" over which the elastin is laid. Elastin: The amorphous, elastic material. Elastin is resistant to degradation, except by elastase. Desmosine and Isodesmosine: Cross-link elastin, forming a network, and stabilizing the elastin during stretching and compressing. Synthesis: First, microfibrillar protein lays down the scaffolding. Then, elastins get laid down on top. AGING: Wrinkles occur as microfibrillar structure is lost Emphysema: Loss of elasticity in lung. Rare form = congenital malfunction of elastase in lung. GROUND SUBSTANCE: Proteoglycans. They consists of a core protein + Glucosaminoglycans Glycosaminoglycans (GAGs): Linear polymers of repeating disaccharides of hexosamine plus a uronic acid such as glucuronic acid. GAG-residues are often sulfated. SIGNALING FUNCTION: GAGs have a high negative charge and are highly hydrophilic. Basic Fibroblast Growth Factor (BFGF) can bind to proteoglycans to promote the growth of fibroblasts. In this capacity proteoglycans also act as a sieve controlling passage of materials through the ECM. This property is especially important in the kidney. Aggrecan: Found in Hyaline Cartilage. Perlecan: Found in Basement Membrane Syndecan: Found in Epithelial Tissue. It remains attached to the plasma membrane. Hyaluronic Acid: Not associated with a core protein itself, but other proteoglycans can associate with it. Tissue Distribution: Vitreous humor of eye. Synovial Fluid of joints. It facilitates cell migration during growth and repair. Hyaluronidase is secreted when hyaluronic acid is no longer needed. BASEMENT MEMBRANES: Made of the Basal Lamina + Reticular Lamina, or two layers of basal lamina. It is visible at the light microscope level, while basal lamina by itself is not. Basal Lamina: It provides a substrate for epithelial cells. It consists of different components:

www.usmle.tv

www.usmle.tv
Lamina Rara: Primary constituent of the basal lamina, composed of two proteins -- laminin and fibronectin. It is directly adjacent to the epithelial cells. It is electron lucent in the electron-microscope. Laminin: Very large protein it three chains. There are specific binding domains for collagen and heparin. Entactin is often associated with Laminin. CANCER: Laminin will hook to integrin receptors. In addition it may have its own receptor, which acts in tumor metastasis. Fibronectin: Two chains. It is important for wound healing and cell migration. There are three forms of fibronectin: Plasma Fibronectin: Binds fibrin and fibrinogen, and plays a role in blood clots. Cell Surface Fibronectin Matrix Fibronectin -- insoluble matrix fibrils. Again, it has specific binding domains for heparin and collagen, and it will hook into cellular integrin receptors. Lamina Densa: The next layer, underneath the Lamina Rara. Composed mainly of Collagen IV (basement membrane collagen) and Heparin. It is electron-dense in the EM microscope. Again, Collagen IV still has its globular registration peptide, so it forms meshworks instead of fibers. Heparin Sulfate interacts electrostatically with the Collagen IV. Lamina Reticularis: The next layer down. Composed of Collagen III and Collagen VII. This makes up the Reticular (elastic) fibers in some basement membranes. Collagen III is the main reticular collagen. Collagen VII acts as an anchor, to hold the reticular fibers to the basal lamina. FNXN: The reticular lamina connects the basal lamina to the underlying stroma. Basement Membrane: The very bottom layer of the epithelial layer. Integrins: Epithelial Cellular receptors that allow the cells to interact with the basement membrane. STRUCTURE: Integral membrane heterodimeric proteins, with alpha and beta subunits non-covalently linked. Ligand-binding Domain: Binds to a specific sequence on laminin and fibronectin in the extracellular matrix. The specific sequence is Arg-Gly-Asp (RGD) Intracellular Attachment: The protein is attached to the actin cytoskeleton, via the following anchor proteins: Talin Vinculin alpha-Actinin FUNCTION: Integrins mediate cellular adhesion and migration through the ECM. LEUKOCYTE MIGRATION: Part of the inflammatory response. Selectins: Specialized glycoproteins on endothelial cells, that serve to attract leucocytes to that location when activated. They allow for stronger interaction of the ECM with the leucocyte integrins. C:ell Adhesion Molecules (CAMs) After being attracted by selectins, the leucocytes interact with CAMs on the endothelial surface. The leucocytes binds to the endothelial cell CAMs.

www.usmle.tv

www.usmle.tv
Activated leucocytes must then secrete proteases and collagenases to migrate through the vessel wall and go to the site of infection. WOUND HEALING: Plasma Fibronectin binds to the blood clot, thus causing Platelet Derived Growth Factor to be released by the platelets. PDGF, along with C5a, then attract neutrophils and macrophages. Macrophages then secrete proteolytic enzymes for fibroblasts and smooth muscle cells, so they can get through the debris. Then the matrix is restored by fibroblasts, then the endothelial cells are restored. TUMOR METASTASIS: Some tumor cells secrete collagenase, thus breaking down basement membranes and allowing the metastatic cells to penetrate the blood vessels. FIBROBLASTS: RESIDENT (always present) Connective tissue cells that synthesize collagen, elastin, and basal lamina. Fibroblasts are not the only cells that synthesize this stuff. Epithelial tissues and smooth muscle cells can make their own ECM, too! Histology: They have little cytoplasm and lots of ER and Golgi, which is what we'd expect for their synthetic roles. Fibroblast Activating Factor up regulates ECM production in fibroblasts. Lymphocytes and monocytes can secrete fibroblast activating factor toward this end. ADIPOCYTES: A RESIDENT CELL in connective tissue -- i.e. it is always present. White Adipose Tissue: Efficient, low-density storage form for energy. It is highly vascularized and innervated. HISTOLOGY: Big lipid droplet with nucleus plus minimal cytoplasmic components all off to one side. Lipid Deposition (Anabolic): Lipoprotein Lipase frees two of the three fats from triacylglycerols from chylomicrons in the blood. The lipoprotein lipase is located in the vascular endothelium. The remaining monoacylglycerol stays in the blood and goes back to liver. The two freed fatty acids diffuse through the capillary endothelium -----> basal lamina ------> connective tissue ------> adipose basal lamina ------> adipocyte ------> and into the adipose tissue. Lipid Mobilization (Catabolic): Hormone Sensitive Lipase is activated via the beta-adrenergic pathway. It frees fatty acids from triacylglycerols in the adipose tissue. beta-Adrenergic Pathway means that Hormone Sensitive Lipase is phosphorylated to be activated (via cAMP ------> Protein Kinase, etc.) Brown Adipose Tissue: Specialized for thermoregulation. It is present in hibernating and newborn humans, but not in human adults. Uncoupling Protein uncouples the oxidation of Acetyl-CoA in adipocyte mitochondria, such that no ATP is produced. Instead, the generated electrochemical gradient is dissipated as heat. OBESITY: Hyperplasia of adipocytes occurs after birth, but the adult doesn't gain or lose adipocytes appreciably. Obesity occurs by hypertrophy of adipocytes. Body Mass Index = (Weight (kg)) / (Height (m))2

www.usmle.tv

www.usmle.tv
Healthy BMI is 23-28. Two forms of obesity: Android Obesity, weight in upper body and abdomen, is correlated with risks for CHD. Gynecoid Obesity, weight in hips and thighs, is not correlated with risks for CHD. "Reduced Obese": When an individual gains weight and then loses it again, several things change physiologically which make it difficult to keep off the weight: Metabolic needs go down from the original baseline level -- i.e. total daily caloric requirements go down after having lost weight. Upregulation of adrenoreceptors occurs -- making it easier to mobilize fatty acids from adipose tissue (that is actually good news). BUT, there is a decreased response to hypoglycemia -- catecholamines aren't released as readily. LEPTIN: A protein made by adipocytes that correlates with obesity in laboratory mice EXPTs in mice suggested that obesity might be due to a lack of leptin. Mice that were obese had no leptin. Unfortunately this did not hold the same for humans. Obese humans actually had more leptin, so there was a positive correlation. There appears to be Leptin receptors in the hypothalamus, which will be involved with hunger regulation. They have also found leptin receptors in the choroid plexus of ventricles. ADIPSIN: It forms Acyl-Stimulating Protein (ASP) which generally promotes the building of triacylglycerols. Many obese patients have elevated adipsin levels, meaning that they can make fats readily but they have normal or subnormal rates of mobilizing them. Tumor Necrosis Factor: Obese patients also seem to have elevated levels of this factor. This is related to development of insulin resistance. MAST CELLS: TRANSIENT Connective Tissue Cell. They function in allergic reactions. They respond to IgE from plasma cells. Histology: They characteristically have cytoplasm full of dark-staining granules. Mast Cell Granules are released in an allergic reaction. They contain: Heparin, an anticoagulant. Histamine, vasodilates small vessels, causing increased microperfusion of the tissue (i.e. redness) Serotonin Leukotrienes MACROPHAGES: TRANSIENT Connective Tissue Cell. They are derived from monocytes circulating in the blood. Phagocytosis is often mediated by IgG Histology: Can be distinguished from other transient cells because they usually have foreign materials ingested. They have numerous small lipid droplets (vacuoles) PLASMA CELLS: TRANSIENT Connective Tissue Cell. They secrete antibodies. Morphology / Histology: They have a clock-face nucleus. They have a perinuclear clear area.

www.usmle.tv

www.usmle.tv

Ø

EMBRYOLOGY v Germ layer Subdivision Derivatives Epidermis ---------------Skin glands, hair, nails Nasal epithelium Ectoderm Surface Oral epithelium and tooth enamel Adenohypophysis Lens of eye, cornea Inner ear Neural tube Brain: neurohypophysis, cranial motor nerves, epiphysis, optic nerve and retina Spinal cord: spinal motor nerves Cranial crest derivatives: sensory ganglia, parasympathetic ganglia, glial and Schwann cells, leptomininges, melanocytes, carotid body and parafollicular cells, many bones of face and cranium, visceral cartilages (throat), connective tissue, minor muscles, carotid body, odontoblasts, thyroid, parathyroid, thymus, salivary and lacrimal glands, outflow tract of heart, cardiac semilunar valves, walls of aorta and aortic arch derived arteries, ciliary muscles, cartilage of external ear Neural crest Trunk crest derivatives: spinal ganglia, parasympathetic ganglia, parasympathetic ganglia, satellite and Schwann cells, melanocytes, adrenal medulla ------------------------------------------------------------------------------Mesoderm --------Somites(Paraxial) Connective tissue of skin Skeletal muscles Axial skeleton Intermediate Kidneys Genital structures Renal and genital ducts Lateral Somatic: connective tissue of ventral body wall, parietal peritoneum, blood vessels, limbs Splanchnic: adrenal cortex, visceral peritoneum, heart, blood vessels ------------------------------------------------------------------------------Endoderm ---------Digestive tube Respiratory epithelium Digestive glands Pharyngeal glands Eustachean tube and lining of middle ear Urinary bladder HIGHLIGHTS OF THE EMBRYONIC PERIOD: (WEEKS 3-8)

www.usmle.tv

www.usmle.tv
---------------------------------3rd week-- primitive streak, notochord, gastrulation, neural folds, early somites 4th week --closure of neuropores, somites, otic pits, branchial arches, limb buds, tail, folding begins, heart is distinct 5th week --growth of head, cervical sinus, hand plates, optic cup 6th week -- external acoustic meatus, prominent cervical flexure, digital rays in hand 7th week-- digital rays in foot, umbilical herniation, yolk sac 8th week-- unquestionably human, genitalia indistinct At the end of 8th week all systems are formed Monthly Periods of Fetal Development -----------------------------------Developmental events of the third through the ninth months (week 9 through week 38). Weeks 9-12--- urine formed, genitalia distinct, hepatic erythropoiesis ends, head is 1/2 size of fetus, fine hair appears Weeks 13-16-- ovaries differentiated, splenic erythropoiesis begins, eyes, ears, and nose almost normal appearance, bile is secreted Weeks 17-20--- fetal movements (quickening), vernix caseosa, lanugo, brown fat, uterus and testes form, fetal heartbeat can be heard with a stethoscope. Weeks 21-24--- respiratory system begins surfactant production, rapid eye movements, skin is wrinkled, blink-startle reflex, fingernails, surfactant Weeks 26-28--- respiratory system matures, splenic erythropoiesis ends, eyelids separate Weeks 29-32--- pupillary reflex Weeks 33-38 --testes descend, bone marrow hematopoiesis, fetus orients to light v Recall: Derivatives - review: 1-derivatives of telencephalon:cerebral Hemispheres,Hippocampus,Amygdaloid,Putamen,Olfactory bulbs,Claustrum,CAUdate,Lamina terminalis:HHAPOCCL:HHeredity Adenomatous POlyposis CCAULi 2-derivatives of diencephalon:thalamus,subthalamus,hypothalamus,epithalamus(every word with thalamus!) Neurohypophysis,Mamillary bodies,pineal Gland,Globus pallidus:NMGG(NeuroMascular GGunction!) iris,ciliary body,retina,(optic nerve,chiasm and tract):all related to eye! Ø Urinary System Questions The pronephros is the first kidney and is developed from intermediate mesoderm, somites 7-14. It is vestigal in humans. The mesonephros is the second kidney, and functions in fetal life before the metanephric kidney is formed and functioning. It’s duct, the mesonephric duct, forms as an extension of the pronephric duct. The mesonephric duct eventually becomes the vas deferens in the adult male, while the mesonephric tubules become the efferent tubules. The ureteric bud gives rise to the collecting system of the metanephric kidney, while the cap gives rise to the filtration portions (nephrons). Both of these arise from intermediate mesoderm.

www.usmle.tv

www.usmle.tv

Unilateral renal agenesis will cause the other kidney to become much larger, while bilateral agenesis will lead to oligohydramnios. The increased pressure and decreased volume of amniotic fluid in the placenta will cause Potter’s fascae, and pulmonary hypoplasia. Early bifurcation of the ureteric bud will cause bifed ureter, while late bifurcation will cause bifed pelves into a single kidney. An extra ureteric bud will lead to a supernumerary kidney or a large, fused kidney. Ectopic ureters in the bladder open along the edge of the trigone region. They can sometimes open into the vagina or urethra, causing incontinence. The ureter feeding the superior kidney enters into the inferior, ectopic position. Multicystic kidneys lead to incomplete development of calyxes and primitive ductal development. Polycystic kidneys are caused by a recessive genetic disorder, leading to problems with differetiation of renal cells, not problems with collecting portions of the kidney. A horseshoe kidney is caused by problems with ascension and rotation of the kidney. It results in a large, fused kidney present in the pelvis. It’s ascent can by blocked by the inferior mesenteric artery, leading to problems. Failure to ascend can also lead to ecotopic kidney, present in the pelvis. Usually this causes no problems. The renal blood supply ascends along with the kidney, and new arteries are created as older ones more caudally are reabsorbed. In the case of a pelvic or horseshoe kidney, arteries will arise from the lower part of the aorta. Ureterocoele occurs when ectoderm from the UG sinus overgrows and occludes the ureteric opening. This tends to occur after the mesonephric ducts have been absorbed into the bladder, creating the trigone region, which is made from intermediate mesoderm. The rest of the bladder is made from splanchnic mesoderm, and the entire thing is lined with endoderm. When this endoderm overgrows the ureteric openings ureterocoele occurs. Posterior urethral valves occur when mucosal flaps occlude the urethra near where the vas enters, causing occlusion. The bladder forms from the cloaca after is partitioned by the urorectal septum, and is made from splanchnic mesoderm. The trigone comes from the absorption of the mesonephric ducts by the bladder. The urachus is the remnant of the allantois, and connects the bladder to the umbilicus. It becomes the median umbilical ligament, but if it remains patent it can cause cysts, fistula, and sinuses. Extrophy of the bladder is caused by a defect in the ventral wall, exposing the trigone portion of the bladder. It is caused by nonclosure of the umbilicus due to a defect in mesodermal migration. It is associated with epispadias because the dorsal surface of the penis does not form properly.

Limb Development Questions

www.usmle.tv

www.usmle.tv

A limb bud is an initial outgrowth from the embronic flanks, induced by FGF-10. Later, the apical ectodermal ridge takes over, promoting growth by releasing FGF-8 and later a number of FGF factors maintaining the limb bud. The zone of polarizing activity is present in the posterior mesoderm, and causes the differential growth of the limb bud in the anterior/posterior plane by releasing sonic hedgehog (Shh) protein. Shh is maintain by FGF from the AER. This causes differences in expression of various HOX genes, shifting their “stripes” from proximal/distal to a posterior/anterior direction. I have no idea what the hell Rindler is talking about here. A child with mirror-image development of the foot could be explained by the presence of two ZPA’s (??), or possibly a ZPA that was somehow shifted to a dorsal/ventral axis (??). The innvervation pattern of the dermatomes reflects the origin and growth of the limb buds in that they show a rotation pattern. Basically, the induction of the limb (whose bones come from lateral mesoderm, and muscles from myotomes) is done by FGF-10, followed by P/D grwoth maintained by the AER releasing FGF-8 and other FGF’s. P/D differentiation occurs as a result of the clock mechanism of the progress zone and it’s release of Msx (only found in the progress zone). A/P differentiaton occurs because of Shh released from the ZPA, which causes stuff to happen with HOX genes. D/V differentiation results from the effects of WNT7a produced by the dorsal ectoderm and Engrailed-1 produced by the ventral ectoderm. Dorsal mesenchyme creates LMX1b, and leads to the formation of the nails and patella.

Hindgut Questions The hindgut gives rise to the distal ½ of the transverse colon (Intraperitoneal), the descending colon (Retroperitoneal), sigmoid colon (I), rectum (R), and anal canal (R). The pectinate line is important because it is the dividing point between endoderm (from the former cloaca) and ectoderm, causes abrupt changes in NV supply and types of muscle. The vascular supply changes from IMA to the inferior rectal arteries, the innervaton changes from autonomic (via symps from lumbar splanchnic nerve, inferior mesenteric ganglion, superior hypogastric plexus, inferior hypogastric plexus, parasym from pelvic splanchnics to inferior mesenteric plexus to synapse in the rectal wall) to somatic (via the inferior rectal branch of the pudendal nerve). Lymphatic drainage changes from the IMA lumbar nodes to inguinal. Abnormalities of hindgut formation: Failure of the Rathke folds causes the UR septum near the anal membrane to remain open, forming a fistula. In males, a rectoprostatic urethra

www.usmle.tv

www.usmle.tv
fistula forms, while in females a rectocloacal fistula can result. Rectovaginal or anovestibular fistula can also result. Basically, the fistula will occur somewhere near the anal membrane. Failure of both the Rathke folds and the UR septum leads to a rectovesical septum in males (because there is no septum, the rectum will join with the bladder, which is formed from the cloaca) and in females, paired vaginas can result. Malalignment of the folds leads to rectoprostatic urethra in males, just as in the failure of the Rathke folds, but also causes penile urethral stenosis, and in females causes a rectovaginal fistula. Imperforate anus occurs when the anal membrane does not rupture. Anal stenosis occurs if it partially ruptures. Other types of abnormalities include low and high defects. Low defects include problems with the protocdeum and anal membrane. Imperf. anus in an example, as in anal agenesis, in which there is no the anus. If a fistula is present, it empties into the urethra, while if there is no fistula the rectum ends blindly. The anus can also be covered by genital folds in what is known as “covered anus”. High anorectal defects – in anorectal agenesis, there is no rectum, anal canal, or anus, and fistulas form into the urethra or vagina. Main difference with rectal defects is that there is no anus whatsoever. In rectal atresia, lack of blood supply causes the rectum to end. However, you still have an anus. In persistent cloaca, the bladder, vagina, and rectum form one cavity. Hirschsprung’s disease is caused by lack of migration of neural crest cells into the colon. Above a certain point, peristalsis occurs, but below that point there is an stenosis that causes a backup of flow, causing the megacolon. The distal aganlionic bowel is narrowed.

Genital Development Questions Where do the primordial germ cells come from? Primordial germ cells come from the epiblast, then migrate to the extraembryonic yolk sac where they remain for 5 weeks. They then migrate through the dorsal mesentery to the posterior body wall next to T10. They are drawn by chemotrophic agents. The coelemic tissue is stimulated by the germ cells to differentiate into the primary sex cords, which gives rise to the genital ridge. Lateral to the genital ridge, the paramesonephrics ducts develop, meeting with the mesonephric ducts inferiorly. In the male, the primary sex cords first give rise to the Sertoli cells, as a result of expression of the SRY gene. These Sertoli cells join with the primordial germ cells to develop into the testis cords. Leydig cells line between the testis cords and secrete testosterone. The Sertoli cells secrete MIF, which causes the degeneration of the paramesonephric duct in males. These testis cords will not become seminiferous tubules

www.usmle.tv

www.usmle.tv
until puberty, when high levels of testosterone cause their differentiation. In the female, the primary sex cords degenerate, and are replaced by the secondary sex cords, which arise from the coelemic epithelium. They surround the primordial germ cells, eventually becoming the follicular cells while the germ cells become oocytes. The secondary sex cords are also called the cortical sex cords, which should give some idea as to what their function is (think of the cortical granules). The function of the SRY gene is to induce the expression of other sexdetermining genes, the most important of which is TDF, or testis determinging factor. So again, SRY leads to the expression of TDF, which in turn induces other genes such as SOX-9. SRY causes all the changes responsible for becoming male – if it deleted, female is the default phenotype. The mesonephric duct leads to the creation of the vas deferens in males (it acquires a layer of smooth muscle), while in the female the mesonephric ducts become the paroophoron and epoophoron. The paramesonephric duct becomes the ovary, uterus, cervix, and upper part of the vagina in the female. The distal part of the paramesonephric ducts fuse together in the midline near the paramesonephric tubercle, an outgrowth of ectoderm. The paramesonephric tubercle leads to the sinovaginal bulbs, which create the vagina. The hymen is a barrier between the vagina and UG sinus. In the male, the paramesonephric ducts become the utricle of the prostate and the appendix of the testis. It degenerates in males due to the effects of MIF. The uterus is formed from the fusion of the paramesophric ducts. The vagina is formed from the vaginal plate, which is formed from the vaginal bulbs. If the ducts don’t fuse properly, a bihorned uterus, or even a double uterus can occur. If the vaginal bulbs don’t fuse, you get a double vagina, or no vagina if the vaginal bulbs don’t develop. The labioscrotal folds form the labia majora in the female, and the scrotum in the male. The folds fuse in the male due to the presence of DHT, which is converted from testosterone by 5-alpha-reductase. DHT also causes the formation of the prostate and the elongation of the phallus, including the fusion of the urethral folds to form the penile urethra. The penile urethra is derived from endoderm, with the exception of the fossa navicularis, an ingrowth of ectoderm. It is formed by the fusion of urethral folds and canalization of the urethral plate. The prostate is formed from the urethra, induced by DHT. It is an endodermal outgrowth. Ø Derivatives of the prosencephalon, mesencephalon, and rhombencephalon What are the derivatives of answer? - Ans. procen.........telen cephalon,dien - mesen..........mesencephalon - rhomben........metenand, myelencephalon - telen......cerebral.h,basalganglia

www.usmle.tv

www.usmle.tv
dien.......thalamus and hypothalamus mesen.........midbrain meten........pons,cerebellum myelen...........medulla Ø Derivatives of the first to fourth POUCHES What happens if no third and fourth pouch? - Ans. 1 => epithelial lining of auditory and middle ear cavity - 2 => lining of palatine tonsil crypts 3 => inf. parathyroid gland, thymus 4 => sup.parathyroid gland absence of 3rd and 4th then no THYMUS AND PARATHYROID, so DiGeorge's Syndrome. Ø Derivatives of the first, second, third, fourth, and sixth arches? If anyone has a mnemonic for this one that would be great! - Ans. Each Arch (mesoderm) is associated with a nerve, (from ectoderm that grows into the arch) gives rise to either arteries, muscles or both, and is also associated with a skeletal structure (from neural crest)... - ARCH 1 (all M's) nerve: mandibular (V-3) artery: none muscle: muscles of mastication and tensor tympani skeletal malleus (and incus) maxilla and mandible - ARCH 2: (all S's) nerve: seven (VII) artery: none muscle: stapedius, stylohyoid, and seven's muscles (facial expression) skel: leSSer horn and upper body of hyoid - ARCH 3: (3=2+1: 2 types of arteries, and 1 muscle) also: 3x3=9: nerve is CN IX) nerve: XI arteries: r and l CC, r and l Internal carotid muscle: stylopharangeus skel: greater horn and lower body of hyoid - ARCH 4 and 6: (nerve is 6+4= 10) - ARCH 4: nerve: X and superior laryngeal artery: r subclavian and arch of aorta muscle: cricothyroid skel: thyroid cart - ARCH 6: nerve: X and recurrent laryngeal artery: R and L Pulm arteries, and Ductus Art. muscle intrinsic muscles of crycothyroid skel: all other laryngeal cart. Ø Derivatives of the first groove? other grooves? What happens if second, third and fourth grooves persist? - Ans. 1....lining of ext.auditory meatus if persisits leads to pharyngeal cysts - first groove, and rest move! (only the first one persists and develops into the lining of the EAM) if the other ones do, that leads to branchial cyst and lateral cervical cyst)

www.usmle.tv

www.usmle.tv
Ø Are the following made from endo, meso or ectoderm? grooves? arches? pouches? - Ans. arches...mesoderm grooves....ecto pouches....endo - way to remember... GAP from out to in: ecto, meso, endo Ø What does the left horn of the sinus venosus develop into? what about the right horn? - Ans. left...coronary sinus rt...smooth part of rt atrium Ø What cranial nerve is associated with pharyngeal pouches I, II, III, IV, and VI? I - V-3 II - VII III - IX IV - X VI - X Ø Where does the foregut, midgut and hindgut end? - Ans. Foregut - upper duodenum midgut - proximal 2/3 of transverse colon hingut - upper part of anal canal Ø What is the adult structure found in the embryo as: left umb. vein? ductus venosus? ductus arteriosus? umb. artery? - Ans. left umbilical v. - lig. teres ductus ven. - lig. venosum ductus arteriosus - lig. arteriosum umbil. art. - medial umbilical ligaments Ø When does the septum primum and septum secundum fuse? before or after birth? and what happens if it does not?\ - Ans. foramen ovale is b/w septum primum and separates secundum. FO fuses after birth, otherwise - left-to-right shunt (ASD?) Ø What are the 5 derivatives of the ventral mesentery? - Ans. Lesser omentum- Hepatoduodenal ,Hepatogastric ligament - plus falciform lig.,coronary lig., triangular lig. - Right... all the liver ligaments are from ventral mesentery (falciform, hepatoduodenal, hepatogastric, coronary, and triangular) All others (gastro-, spleno-, SI, LI, and colon) are derived from Dorsal mesentery Ø What three things cause the indifferent gonad to become a testis? and where do they come from? - Ans. Testostrone by Leydig MIF: by Sertoli and finally the main TDF on Y chromosome (short arm)

www.usmle.tv

www.usmle.tv

Ø What are the arteries of the foregut, midgut and hindgut? What is the only organ supplied by the foregut artery, that is of mesodermal origin? - Ans. celiac, superior mesenteric, inferior mesenteric. - Spleen is a mesodermal organ and gets blood supply from celiac artery Ø What structure is derived from the prechordal plate? - Ans. MOUTH.

www.usmle.tv

Sponsor Documents

Or use your account on DocShare.tips

Hide

Forgot your password?

Or register your new account on DocShare.tips

Hide

Lost your password? Please enter your email address. You will receive a link to create a new password.

Back to log-in

Close